Anda di halaman 1dari 97

MA13 – Geometria – AV1 – 2014

Questão 1 [ 2,0 pt ]

Considere um paralelogramo ABCD e sejam


• M o centro da circunferência definida pelos vértices A, B e C
• N o centro da circunferência definida pelos vértices B, C e D;
• O o centro da circunferência definida pelos vértices C, D e A;
• P o centro da circunferência definida pelos vértices D, A e B;
com M , N , O e P dois a dois distintos. Nessas condições, e sabendo que ABCD não é um retângulo,
mostre que:
(a) M N OP é um paralelogramo;
(b) M N OP e ABCD possuem ângulos internos iguais.

Solução

(a) Como M é centro de uma circunferência que contém A e B, temos M A ≡ M B, portanto, M pertence à
mediatriz do lado AB. Mas P também é centro de uma circunferência que contém A e B, logo, P também
pertence à mediatriz do lado AB. Assim, M P está contido nesta mediatriz.

Da mesma forma, N e O são centros de circunferências que contêm os pontos C e D, portanto, N e O pertencem
à mediatriz do lado CD, o que implica que o segmento N O está contido nesta mediatriz.
Como ABCD não é um retângulo, as mediatrizes que contêm M P e N O não coincidem. Como os lados AB
e CD são paralelos, suas respectivas mediatrizes também serão. Mas, como os segmentos M P e N O estão
contidos cada um em uma destas duas mediatrizes, eles serão também paralelos. Por argumentação análoga,
M N e OP estão contidos, respectivamente, nas mediatrizes de BC e AD, portanto serão também paralelos.
Isto mostra que M N OP é um paralelogramo.
(b) Denotemos por E e F os pontos médios de AB e BC, respectivamente.

Considerando o quadrilátero M EBF , temos

∠(EBF ) + ∠(BF M ) + ∠(F M E) + ∠(M EB) = 360◦ ,

e, como B F̂ M e M ÊB são retos, temos

∠(ABC) + ∠(F M E) + ∠(EBF ) + ∠(F M E) = 180◦ .

Por outro lado,


∠(ON M ) + ∠(F M E) = ∠(ON M ) + ∠(N M P ) = 180◦ ,

pois ON̂ M e N M̂ P são ângulos suplementares, já que M P e OP são retas paralelas.
Assim, ∠(ABC) = ∠(ON M ), valendo o mesmo para seus congruentes no paralelogramo, isto é, ∠(ADC) =
∠(M P O). Consequentemente, seus suplementares B ĈD, DÂB, N M̂ P e P ÔN serão também congruentes.

Questão 2 [ 2,0 pt ]

Seja ABCD (em ordem) um quadrilátero convexo. Prove que se

∠BDA = ∠ACB
então o quadrilátero é inscritı́vel em um cı́rculo. Verifique, justificando, se a recı́proca é verdadeira.

Solução
Considere o cı́rculo determinado por A, B e C. Vamos provar que D pertence a esta circunferência. Seja D0 a
−−→
interseção (diferente de B) da semirreta BD com a circunferência. Supondo por absurdo que D não pertença à
circunferência, teremos D0 6= D. Com isso, ∠(BD0 A) = ∠(ACB), pois ambos são ângulos inscritos na circunferência
_
e determinam o mesmo arco AB, e, com isso, ∠(BD0 A) = ∠(BDA), pois ∠(BDA) = ∠(ACB) por hipótese.
Caso D esteja entre D0 e B, o ângulo externo AD̂B do triângulo ADD0 seria congruente ao ângulo interno AD̂0 D,
o que é um absurdo. Caso D0 esteja entre D e B, o ângulo externo AD̂0 B seria congruente ao ângulo interno AD̂D0 ,
o que também é um absurdo.
A recı́proca é obviamente verdadeira, pois se ABCD é inscritı́vel em um cı́rculo, os ângulos inscritos B D̂A e AĈB
_
determinarão o mesmo arco AB, portanto, ∠(BDA) = ∠(ACB).

Questão 3 [ 2,0 pt ]

Num cı́rculo, tomam-se um diâmetro AB e uma corda CD, perpendiculares entre si. Sendo E um ponto
qualquer de CD, as retas suportes de AE e BE cortam a circunferência em F e G, respectivamente.
−→ −−→
(a) Prove que a semirreta F A bissecta C F̂ D e que a semirreta GB bissecta C ĜD.
(b) Prove que o quadrilátero CF DG tem dois lados consecutivos na mesma razão que os dois outros.

Solução

(a) Como AB é um diâmetro que intersecta a corda CD perpendicularmente, esta interseção ocorrerá no ponto
médio M de CD. Assim, sendo O o centro do cı́rculo, pelo caso LLL, os triângulos COM e DOM serão
congruentes, e então C ÔA ≡ DÔA.

_ _
Com isso, AC≡AD, portanto os ângulos inscritos que os determinam, C F̂ A e AF̂ D, serão congruentes. Isto
−→
mostra que a semirreta F A intersecta o ângulo C F̂ D.
_ _
Da mesma forma, BC≡BD, logo os ângulos incritos, C ĜB e DĜB, serão congruentes. Isto mostra que a
−−→
semirreta GB intersecta o ângulo C ĜD.

(b) Considerando o triângulo CF D, F E será uma bissetriz interna, portanto

CE DE DF DE
= ∴ = .
CF DF CF CE

Da mesma forma, no triângulo CGD, GE também será uma bissetriz interna, e então

CE DE DG DE
= ∴ = .
CG DG CG CE

Assim,
DF DG
= .
CF CG
Questão 4 [ 2,0 pt ]

Dado um triângulo acutângulo ABC de baricentro G, constrói-se os triângulos equiláteros ABD e ACE,
exteriores a ABC, de baricentros H e I, respectivamente.
(a) Mostre que CD ≡ BE.
(b) Mostre que GH ≡ GI.

Solução

(a) Observe que ∠(CAD) = ∠(CAB) + 60◦ = ∠(EAB). Além disso, como AC ≡ AE e AD ≡ AB, pelo caso LAL,
os triângulos ACD e AEB serão congruentes. Com isso, os lados BE e CD serão congruentes.

(b) Sejam M e N pontos médios de AB e AC, respectivamente.

Como
1 1
GN = BN e IN = EN ,
3 3
e como GN̂ I ≡ B N̂ E, os triângulos BN E e GN I serão semelhantes, de razão de semelhança 31 . Assim,

1
GI = BE.
3
Analogamente, trabalhando com os triângulos semelhantes CM D e GM H,
1
GH = CD.
3

Mas, como CD ≡ BE, temos GI = GH, portanto GI ≡ GH.


Questão 5 [ 2,0 pt ]

Seja ABCD um trapézio de base maior AB e base menor CD. Sendo AB = a, CD = c, BC = b, AD = d,


determine a soma dos quadrados das medidas das diagonais AC e BD em função de a, b, c e d. Solução

Sejam por P e Q as projeções ortogonais sobre AB de C e D, respectivamente. Denotemos por m e n as medidas de


P B e AQ, respectivamente, e por h a altura do trapézio, relativa a AB.

Aplicando o Teorema de Pitágoras aos triângulos AP C e BQD, temos

D12 = (a − m)2 + h2 ,

D22 = (a − n)2 + h2 ,

onde D1 e D2 são as medidas das diagonais AC e BD, respectivamente. Por outro lado, aplicando Pitágoras aos
triângulos AQD e BP C,
h2 = d2 − n2 ,

h2 = b2 − m2 ,

logo,
D12 = (a − m)2 + d2 − n2 ,

D22 = (a − n)2 + b2 − m2 .

Somando as duas equações, temos

D12 + D22 = (a − m)2 + d2 − n2 + (a − n)2 + b2 − m2


= 2a2 − 2am − 2an + b2 + d2
= 2a2 − 2a(m + n) + b2 + d2 .

Como m + n = a − c, temos então

D12 + D22 = 2a2 − 2a(a − c) + d2 + b2 = 2ac + b2 + d2 .


MA13 – Geometria – AV2 – 2014

Questão 1 [ 2,0 pt ]

Na figura a seguir temos que ∠BAC = x/2, ∠BAD = y/2, medidos em radianos, e AB = 2.

Com base nessas informações:

(a) Expresse a área dos triângulos ABC e ABD como funções de x e y.

(b) Mostre que


Área (ABD) Área (AED)
<1+ .
Área (ABC) Área (ABE)
(c) Mostre que para 0 < x < y < π/2 vale
sin(x) sin(y)
> .
x y

Solução

(a) Como AB = 2, considerando o triângulo ABC, retângulo em C, temos

BC BC x
= = sen (∠BAC) = sen ,
2 AB 2
logo x
BC = 2sen .
2
Da mesma forma,
AC AC x
= = cos (∠BAC) = cos ,
2 AB 2
logo x
AC = 2cos .
2
Assim,
x x
 
BC · AC 2sen 2 · 2cos 2
x x
Área(ABC) = = = 2 sen cos = sen(x).
2 2 2 2
Considerando agora o triângulo ABD, retângulo em D, e ainda utilizando que AB = 2, temos

BD BD y
= = sen (∠BAD) = sen ,
2 AB 2
logo y
BD = 2sen ,
2
e
AD AD y
= = cos (∠BAD) = cos ,
2 AB 2
que implica y
AD = 2cos .
2
Então,
y y
 
BD · AD 2sen 2 · 2cos 2
y y
Área(ABD) = = = 2 sen cos = sen(y).
2 2 2 2
(b) Observe que
Área(ABD) = Área(ABE) + Área(AED)

e
Área(ABC) = Área(ABE) + Área(BCE) > Área(ABE).

Assim,
Área(ABD) Área(ABE) + Área(AED) Área(AED)
< =1+ .
Área(ABC) Área(ABE) Área(ABE)
(c) Pelos itens anteriores, temos
sen(y) Área(ABD) Área(AED)
= <1+ .
sen(x) Área(ABC) Área(ABE)
_
O setor circular de centro A e determinado pelo arco F E está contido no triângulo ABE. Como a medida de
_
F E em radianos é x/2, temos
rx
Área(ABE) > ,
2
onde r é o raio do cı́rculo.
_ _
O setor circular de centro A e determinado pelo arco EG contém o triângulo AED. Como a medida de EG
em radianos é y/2 − x/2 = (y − x)/2, temos

r(y − x)
Área(AED) < .
2
Assim,
r(y−x)
sen(y) Área(AED) 2 y−x y y
<1+ <1+ rx =1+ =1+ −1= ,
sen(x) Área(ABE) 2 x x x
portanto
sen(x) sen(y)
> .
x y

Questão 2 [ 2,0 pt ]

Considere três retas r, s e t do espaço tais que qualquer plano seja concorrente a pelo menos uma destas
retas. Considere ainda um poliedro tal que

• todas as suas faces são quadriláteros;


• cada uma de suas arestas é paralela a alguma das retas r, s ou t; e

Prove que todas as faces deste poliedro são paralelogramos.

Solução
Considere uma das faces do poliedro, a qual chamaremos de ABCD. Sem perda de generalidade, digamos que
AB k r. Não poderemos ter BC k r, pois, neste caso, teremos AB e BC consecutivos e colineares. Digamos então,
sem perda de generalidade, que BC k s.
Observe que já concluı́mos que cada uma das retas r e s é paralela ou está contida no plano da face ABCD. Assim,
pela informação dada sobre as retas, t deve ser concorrente a tal plano, logo não pode ser paralela a este plano nem
pode estar contida nele. Portanto, nenhuma das arestas CD e AD desta face pode ser paralela à reta t.
Não poderemos ter CD k s (pois, nesse caso, BC e CD seriam colineares) nem, como já vimos, CD k t. Portanto,
CD k r. Da mesma forma, AD não pode ser paralela a r, logo AD k s.
Até aqui, já vimos que ABCD é um paralelogramo. Mas a mesma argumentação vale para qualquer uma das faces
do poliedro, portanto todas as suas faces são paralelogramos.

Questão 3 [ 2,0 pt ]


1+ 5
Sabendo que a diagonal de um pentágono regular mede 2 de seu lado, determine o cosseno do ângulo
entre duas faces adjacentes de um icosaedro regular.

Solução
A partir de um vértice A do icosaedro, podemos retirar uma pirâmide de base BCDEF pentagonal regular, cujas
faces laterais são cinco triângulos equiláteros que são faces do icosaedro. Se tomarmos o ponto médio M de AC, o
ângulo entre as faces ABC e ACD será dado por ∠BM D.

Denotando por l a medida da aresta do icosaedro, como BM e DM são alturas de triângulos equiláteros de lado
l, teremos √
l 3
BM = DM = .
2
Como BD é uma diagonal do pentágono regular BCDEF , de lado l, temos

1+ 5
BD = l.
2
Assim, pela lei dos cossenos, fazendo ∠BM D = θ, temos
√ !2 √ !2 √ !2 √ ! √ !
1+ 5 l 3 l 3 l 3 l 3
l = + −2 cos θ,
2 2 2 2 2
logo √
3+ 5 3 3
= − cos θ
2 2 2
e então √
5
cos θ = − .
3
Assim, o ângulo entre as faces é
√ !
5
θ = arccos − .
3

Questão 4 [ 2,0 pt ] Questão anulada

Um poliedro convexo tem exatamente 3 faces triangulares, 1 face quadrangular, 1 face pentagonal e 2 faces
hexagonais. Obtenha:

(a) O número total de vértices, faces e arestas do poliedro.

(b) O número de diagonais do poliedro

(c) A soma dos ângulos internos de todas as faces.

Solução
A questão está anulada. O poliedro descrito no enunciado não existe; vejamos o motivo.

Somando o número de faces de cada tipo, o poliedro terá 3 + 1 + 1 + 2 = 7 faces. Multiplicando o número de arestas
em cada tipo de face pelo número de faces de cada tipo, e somando os resultados obtidos, estaremos “contando” duas
vezes cada aresta, assim,
2A = 3 · 3 + 1 · 4 + 1 · 5 + 2 · 6 = 9 + 4 + 5 + 12 = 30,

logo A = 15. Como


V − A + F = 2,

temos
V − 15 + 7 = 2, logo V = 10.

Por outro lado (veja Problema 2.5 do Capı́tulo 9),

30 = 2A = 3V3 + 4V4 + 5V5 + ...,

onde Vi é o número de vértices nos quais incidem i arestas.


Assim,

30 = 3V3 + 4V4 + 5V5 + ... ≥ 3V3 + 3V4 + 3V5 + ... =

= 3(V3 + V4 + V5 + ...) = 3V = 30.

Com isso, a desigualdade é, na verdade, uma igualdade, e, portanto,

3V3 + 4V4 + 5V5 + ... = 3V3 + 3V4 + 3V5 ,


o que só é possı́vel se V4 = V5 = ... = 0, logo V3 = V = 10. Assim, o poliedro possui apenas vértices nos quais
incidem 3 arestas.

De posse dessa informação, vamos tentar construir o poliedro.

Se as duas faces hexagonais não tiverem vértice comum (figura 1), o poliedro deverá ter pelo menos 12 vértices.
Mas isto contraria V = 10.
Se as duas faces hexagonais tiverem apenas um vértice comum (figura 2), o poliedro deverá ter pelo menos 11
vértices, contrariando também V = 10.

Assim, como duas faces não podem ter três ou mais dois vértices em comum, as faces hexagonais terão exatamente
dois vértices em comum (figura 3). Note que os vértices destas duas faces já são todos os 10 vértices que o poliedro
pode ter.
No vértice A acima não pode incidir nenhuma outra aresta (pois nele já incidem as 3 arestas possı́veis). Portanto,
devemos ter A, F e J em uma mesma face. Por outro lado, esta face não pode ter nenhum dos outros pontos como
vértice, pois, nesse caso, ela teria três vértices em comum com alguma das faces hexagonais. Assim, AF J será uma
face (Figura 4).

Da mesma forma, BCG também é uma face (Figura 4).


Repare que F , J, C e G já “esgotaram” as arestas que neles incidem.
O segmento F J é aresta de alguma face, mas como não é possı́vel incidir mais alguma aresta em F e J, esta
face deverá ter como vértices os pontos E, F J e I. Veja que esta face já tem dois vértices em comum com cada
uma das duas faces hexagonais, logo não poderá ter nenhum outro vértice. Assim, tal face é EF JI, logo uma face
quadrangular (Figura 5).
Da mesma forma, devemos ter obrigatoriamente a face DCGH (Figura 5), o que é um absurdo pois, pelo enunciado,
não podemos ter mais de uma face quadrangular.

Questão 5 [ 2,0 pt ]

O sólido da figura é limitado pelo triângulo ABC, pela lateral de um cone de vértice A e por um segmento
circular de centro O. Sabe-se que O é a projeção ortogonal de A sobre o plano que contém o cı́rculo
representado, que o ângulo B ÔC é reto e que OA = 6cm e OB = 3cm. Determine o volume do sólido.

Solução
Como o ângulo B ÔC é reto, o sólido em questão é obtido retirando-se a pirâmide de base BOC e vértice A de um
quarto do cone circular reto, cuja base é o cı́rculo e cujo vértice é A. O volume do quarto de cone será dado, em
unidades de volume, por
2
!
1 π · OB · OA 1 π · 32 · 6 9π
Vc = = · = .
4 3 4 3 2
O volume da pirâmide cuja base é o triângulo retângulo BOC e cuja altura é OA é dado, em unidades de volume,
por
BO·CO 9
2 · OA 2 ·6
Vp = = = 9.
3 3
Assim, o volume procurado é, em unidades de volume,
9π π 
V = Vc − Vp = −9=9 −1 .
2 2
MA13 – Geometria – AV3 – 2014

Questão 1 [ 2,0 pt ]

Sejam P T e P U segmentos tangentes a duas circunferências concêntricas, com T pertencente à menor e U


à maior. Se o segmento P T corta a circunferência maior no ponto Q, mostre que

2 2 2
P T − P U = QT .

Solução
Prolongue QT na direção de T até o ponto R da circunferência maior. Sendo O o centro de ambos os cı́rculos, como
o raio OT é perpendicular a QR, temos que QT = T R A potência de P em relação ao cı́rculo maior é dada tanto

2
por P U quanto por P Q · P R, logo
2
P U = P Q · P R.

Mas P Q = P T − QT e P R = P T + T R = P T + QT . Assim,
2   2 2
P U = P Q · P R = P T − QT P T + QT = P T − QT .

Portanto,
2 2 2
P T − P U = QT .

Solução alternativa
Como P T e P U são tangentes às circunferências, serão retos os ângulos P T̂ O e P Û O. Assim, aplicando o Teorema
de Pitágoras nos triângulos QT O, P T O e P U O, temos
2 2 2
QO = T O + QT , (1)
2 2 2
PO = TO + PT , (2)
2 2 2
PO = UO + PU . (3)
Reescrevendo (1), temos
2 2 2
T O = QO − QT . (4)
Por (2) e (3), temos
2 2 2 2
TO + PT = UO + PU ,
logo, substituindo a expressão para T O obtida em (4), obtemos
2 2 2 2 2
QO − QT + P T = U O + P U .

Como U O e QO são raios de uma mesma circunferencia, e, portanto, U O = QO, a expressão acima nos dá
2 2 2
−QT + P T = P U ,

que, reescrevendo, fica


2 2 2
P T − P U = QT .

Questão 2 [ 2,0 pt ]

Considere um polı́gono P circunscrito a um cı́rculo C. Se uma reta r passa pelo centro de C e divide P em
dois polı́gonos, P1 e P2 , prove que P1 e P2 têm mesma área se, e somente se, têm o mesmo perı́metro.

Solução
Sejam P e Q os pontos onde a reta r intersecta o polı́gono e denote por
• P, A1 , . . . , An , Q os vértices do polı́gono P1 e
• P, B1 , . . . , Bm , Q os vértices do polı́gono P2 ,

como na figura abaixo.


Denotemos ainda

• l1 = P A1 , l2 = A1 A2 , . . ., ln = An−1 An e ln+1 = An Q e

• l10 = P B1 , l20 = B1 B2 , . . ., lm
0 0
= Bm−1 Bm e lm+1 = Bm Q.

Observe que o perı́metro de P1 é dado por

2P1 = l1 + l2 + . . . + ln + ln+1 + P Q,

e o de P2 é dado por
2P2 = l10 + l20 + . . . + lm
0 0
+ lm+1 + P Q.

Decompondo o polı́gono P1 em n + 1 triângulos de altura dada pelo raio r do cı́rculo, que tenham o centro O do
cı́rculo como um dos vértices (figura), a área de P1 é dada por

l1 · r l2 · r ln · r ln+1 · r
Área(P1 ) = + + ... + +
2 2 2 2
(l1 + l2 + . . . + ln−1 + ln )r
=
2
(2P1 − P Q)r
= .
2
Da mesma forma, decompondo o polı́gono P2 em m + 1 triângulos, temos
0
l10 · r l20 · r l0 · r lm+1 ·r
Área(P2 ) = + + ... + m +
2 2 2 2
(l10 + l20 + . . . + lm−1
0
+ lm0
)r
=
2
(2P2 − P Q)r
= .
2
Assim,
(2P1 − P Q)r (2P2 − P Q)r
Área(P1 ) = Área(P2 ) ⇔ = ⇔ 2P1 = 2P2 .
2 2
Questão 3 [ 2,0 pt ]

Sejam ABCD um quadrado de lado L, a semicircunferência de diâmetro CD, o segmento BG tangente à


semicircunferência em E, conforme a figura abaixo. Calcule, em função de L, a medida do segmento DG.

Solução
Seja F o centro da semicircunferência. Como BE ≡ BC, e F E ≡ F C, os triângulos BEF e BCF serão congruentes
pelo caso LLL, logo B F̂ E ≡ B F̂ C.
Da mesma forma, serão congruentes os triângulos GEF e GDF . Logo,

GF̂ E ≡ GF̂ D e DĜF ≡ E ĜF.

Como
180◦ = ∠(DF E) + ∠(EF C) = 2∠(GF D) + 2∠(BF C),

temos
∠(GF D) + ∠(BF C) = 90◦ ,

e, como ∠(CBF ) + ∠(BF C) = 90◦ , temos GF̂ D ≡ C B̂F . Assim, os triângulos retângulos BCF e F DG são
semelhantes, com
L
BC CF L L
= ∴ L = 2 ∴ DG =
FD DG 2 DG 4
Questão 4 [ 2,0 pt ]

Um plano é perpendicular à diagonal AG do cubo ABCDEF GH da figura, de forma que sua interseção
com as faces do cubo seja o hexágono U V W XY Z.

(a) Mostre que cada lado do hexágono U V W XY Z é paralelo a uma das diagonais da face do cubo em
que está contido.

(b) Determine o perı́metro do hexágono U V W XY Z, sendo 1 a medida da aresta do cubo.

Solução

(a) Primeiramente, observe que a diagonal AG do cubo é ortogonal às diagonais de face ED, BD e BE, pois AG
contém a altura do tetraedro AEDB. Da mesma forma, AG é ortogonal às diagonais CH, CF e F H.
O plano π considerado, perpendicular a diagonal AG, é paralelo ao plano determinado por E, D e B, portanto, a
interseção Y Z com a face ADHE é paralela à diagonal ED desta face. Analogamente, U V e W X são paralelos
às diagonais BD e BE das faces em que estão.
Da mesma forma, π é paralelo ao plano determinado por C, F e H, portanto, U Z, V W e XY serão paralelos,
respectivamente, às diagonais de face CH, CF e F H.

(b) Pelo item (a), serão isósceles os triângulos CU V , BV W , F XW , EXY , HY Z e DU Z. Fazendo HY = a,


teremos então
a = HY = HZ = U C = CV = W F = XF ,

1 − a = EY = DZ = DU = BV = BW = EX.

Assim,

Y Z = U V = W X = a 2,

U Z = V W = XY = (1 − a) 2.

Logo, o perı́metro de U V W XY Z é dado por


√ √ √
Y Z + U V + W X + U Z = V W = XY = 3a 2 + 3(1 − a) 2 = 3 2.
Questão 5 [ 2,0 pt ]

Considere o cubo ABCDEFGH de aresta a. Um cone C1 tem base inscrita na face ABCD e vértice na
intersecção das diagonais da face EFGH. Outro cone C2 tem base inscrita na face EFGH e vértice na
intersecção das diagonais da face ABCD. Calcule o volume da parte comum a esses dois cones.

Solução
Seja I o ponto na face ABCD que é vértice de um dos cones, e J o ponto na face EF GH, vértice do outro cone.
Considere os pontos K e M , interseção de cada um dos cones com as arestas AB e EF , respectivamente (veja a
figura).

Como K e M são pontos médios das arestas em que estão, e I e J são os centros de suas faces, IJM K é um
retângulo, cujo centro é o ponto L, interseção das geratrizes IM e JK dos cones. A distância de L ao centro O do
cubo será dada então por KI/2 = a/4. A distância de O a cada um dos pontos I e J será igual a a/2.
Assim, a interseção dos cones será um sólido formado por dois cones, cuja base é um cı́rculo de raio OL = a/4, e
cujas alturas OI e OJ medem a/2. Portanto, o volume deste sólido é dado por
2 !
π a4 · a2 π a3
V =2 = .
3 48
MA13 – Geometria – AVF – 2014

Questão 1 [ 2,0 pt ]

Na figura, AB ≡ AC e a bissetriz interna traçada de B intersecta o lado AC em P de forma que AP + BP = BC.


Os pontos Q e D são tomados de forma que BQ ≡ BP e P D é paralelo a BC.

(a) Mostre que os triângulos CQP e P AD são congruentes.

(b) Determine as medidas dos ângulos do triângulo ABC.

Solução

(a) Para simplificar a notação, denote ∠(CBP ) = β. Como BP é bissetriz de AB̂C, temos também ∠(ABP ) = β. E, como
AB ≡ AC, temos ∠(ACB) = ∠(ABC) = 2β.

Como BQ + QC = BC = BP + P A e BQ = BP , temos QC = AP .
Como BC e DP são paralelas, temos
∠(BP D) = ∠(CBP ) = β,

∠(AP D) = ∠(ACB) = 2β.

Como AD̂P é ângul oexterno do triângulo BDP , temos

∠(ADP ) = ∠(BP D) + ∠(DBP ) = 2β.

Mas então o triângulo ADP será isósceles, e, portanto, AD ≡ AP . Como AB ≡ AC, temos que BD ≡ P C, e, como
BDP é isósceles, temos DP ≡ BD ≡ CP .
Pelo caso LAL, temos então que os triângulos P AD e CQP são congruentes.
(b) Pela congruência de P AD e CQP , temos ∠(QP C) = ∠(ADP ) = 2β.

Como P QC é isósceles, ∠(QP B) = ∠(P QB), e como ∠(QP B) + ∠(P QB) + β = 180◦ , temos ∠(QP B) = 90◦ − β
2
.
Assim, considerando todos os ângulos da figura que têm vértice em P , temos
 
◦ β
2β + 90 − + β + 2β = 180◦ ,
2
logo
9
β = 90◦ ∴ β = 20◦ .
2
com isso, os ângulos internos B̂ e Ĉ medem 2β = 40◦ , e  mede 180◦ − 2 · 40◦ = 100◦ .

Questão 2 [ 2,0 pt ]

Prove que se um trapézio isósceles tem os lados congruentes com comprimento a, os lados paralelos com comprimentos
b e c, e diagonais com comprimento d, então d2 = a2 + bc.

Solução
Vamos supor, sem perda de generalidade, b > c. Denotemos por h a altura do trapézio e por m a projeção ortogonal de um
b−c
dos lados congruentes sobre a base maior. Note que m = 2
.

Assim, considerando um triângulo retângulo cujos catetos são m e h, e cuja hipotenusa é um lado a, temos, pelo Teorema
de Pitágoras,
m2 + h2 = a2 ,
logo
 2
b−c
+ h2 = a2 ,
2
que nos dá
b2 + c2 − 2bc + 4h2 = 4a2 .
Considerando agora um triângulo retângulo cujos catetos medem c + m e h, e de hipotenusa d, temos

(c + m)2 + h2 = d2 ,

logo
 2
b−c
c+ + h2 = d 2 ,
2
e então  
b+c
+ h2 = d 2 ,
2
que implica
b2 + c2 + 2bc + 4h2 = 4d2 .

Subtraindo as duas equações obtidas, temos


4bc = 4d2 − 4a2 ,

logo,
d2 = a2 + bc.

Questão 3 [ 2,0 pt ]

Na figura abaixo o triângulo ABC é retângulo em A. Os quadriláteros ABED e ACGF são quadrados. Estendemos
EB até P , de tal modo que EB ≡ BP . Estendemos GC até Q, de tal modo que GC ≡ CQ.

(a) Prove que o triângulo ABC é congruente ao triângulo P BI e que o triângulo BQC é congruente ao triângulo
HP I.

(b) Prove que a área do triângulo BP C é a metade da área do quadrado ABED.

(c) Prove que a área do triângulo BQC é a metade da área do quadrado ACGF .
2 2 2
(d) Demonstre que AB + AC = BC (Teorema de Pitágoras).

Solução

(a) Por simplicidade, vamos denotar ∠(ABC) = β e ∠(ACB) = γ. Note que β + γ = 90◦ .
Como ∠(ABC) = β e AB̂C é ângulo reto, ∠(P BC) = 90◦ − β = γ. E, como C B̂I é ângulo reto, ∠(P BI) = 90◦ − γ = β.
Assim, temos P B ≡ EB ≡ AB, BI ≡ BC e ∠(P BI) = ∠(ABC), logo, pelo caso LAL, os triângulos ABC e P BI são
congruentes.
Essa congruência implica que ∠(P IB) = ∠(ACB) = γ, logo ∠(P IH) = 90 − γ = β, e que P I ≡ AC.
Temos ∠(BCQ) = 90◦ − γ, logo, ∠(BCQ) = β.
Note ainda que QC ≡ CG ≡ AC.
Temos então que
∠(P IH) = β = ∠(BCQ),

P I ≡ AC ≡ QC,

IH ≡ CB.

Com isso os triângulos P BI e BQP serão congruentes pelo caso LAL.

(b) Como AC é paralelo a BP , temos


2
BP · AB AB
Área(BP C) = Área(BP A) = = ,
2 2
2
que é a metade da área AB do quadrado ABED.

(c) Como AB é paralelo a QC, temos


2
QC · AC AC
Área(BQC) = Área(AQC) = = ,
2 2
2
que é a metade da área AC do quadrado ACGF .

(d) Pelos itens anteriores, já sabemos que


2 2
AB + AC = 2Área(BP C) + 2Área(BQC).
2
Portanto, resta apenas mostrar que 2Área(BP C) + 2Área(BQC) = BC
Pelo item (a), temos Área(BQC) = Área(P IH), sendo h1 e h2 as alturas dos triângulos BP C e P IH, relativas às bases
BC e IH, respectivamente (veja figura), temos h1 + h2 = CH = BC, logo

2Área(BP C) + 2Área(BQC) = 2Área(BP C) + 2Área(P IH)


BC · h1 IH · h2
= 2· +2·
2 2
= BC · h1 + BC · h2
= (h1 + h2 )BC
2
= BC .

Assim,
2 2 2
AB + AC = 2Área(BP C) + 2Área(BQC) = BC .

Questão 4 [ 2,0 pt ]

Um tetraedro regular é cortado por um plano paralelo a duas arestas, de tal forma que a seção seja um paralelogramo.

(a) Descreva a posição do plano de forma que a seção seja um losango e calcule, em função de a, o lado desse
losango.
(b) Determine, em função da medida a da aresta, a medida do lado do paralelogramo de área máxima assim obtido.

Solução

(a) Para facilitar a escrita, seja ABCD o tetraedro e consideremos o plano paralelo às arestas AB e CD. Sejam ainda X,
Y , Z e W as interseções do plano com as arestas AB, BC, BD e AD, respectivamente.

O segmento XY é a interseção do plano paralelo a AB com a face ABC, contida em um plano que também contém
AB. Ora, interseção de dois planos paralelos a uma reta dada (ou que contenham esta reta) será uma reta paralela à
reta dada, portanto, o segmento XY é paralelo à aresta AB. Da mesma forma, o segmento ZW será paralelo a AB, e
Y Z e XW paralelos a CD.
Como ABC é um triângulo equilátero, e XY é paralelo a AB, temos que XCY é equilátero, implicando XY ≡ CX.
Da mesma forma, como ACD é equilátero e XW é paralelo a CD, temos que XW ≡ AX.
Para que XY ZW seja um losango, é necessário que XY ≡ XW , logo, que

CX ≡ XY ≡ XW ≡ AX.

Assim, X será o ponto médio de AC. Com isso, como CY ≡ CX e BC ≡ AC, Y também será ponto médio de BC. Da
mesma forma, Z e W são pontos médios de BD e AD, respectivamente.

(b) Na notação do item anterior, sendo XC = x, teremos XY = x e XW = AX = a − x. As retas reversas suporte de AB


e CD são ortogonais, logo, sendo XY e XW paralelas a estas retas, respectivamente, XY e XW são perpendiculares.
Assim, a área do paralelogramo XY ZW é dada por

Área(XY ZW ) = XC · XW = x(a − x) = x2 + ax.

a
O valor máximo para esta expressão ocorre quando x = 2
. Portanto, o paralelogramo de área máxima ocorre quando
a
seus lados medem 2
.

Questão 5 [ 2,0 pt ]

Sejam x, y e z os volumes gerados por um triângulo ABC, retângulo em A, girando sucessivamente em torno de seus
lados BC, CA e AB. Prove que
1 1 1
= 2+ 2
x2 y z

Solução
Quando giramos em torno do cateto CA, temos um cone circular reto cuja base é o cı́rculo de raio AB e de altura CA.
Assim, o volume é
2
πAB · CA
y= .
3
Quando giramos em torno do cateto AB, temos um cone circular reto cuja base é o cı́rculo de raio CA e de altura AB.
Assim, o volume é
2
πCA · AB
z= .
3
Girando em torno da hipotenusa BC, obtemos dois cones. A base de ambos será o cı́rculo de raio igual à altura h do
triângulo ABC, relativa ao vértice A. As alturas dos cones serão as medidas m e n das projeções ortogonais de AB e AC
sobre a hipotenusa BC. Assim, o volume do sólido dado pelos dois cones é

πh2 · m πh2 · n
x = +
3 3
πh2 (m + n)
=
3
πh2 · BC
=
3
AB·CA
Como h · BC = AB · CA, temos h = BC
, logo
2 2
πAB · CA
x= .
3BC
Assim,
1 1 9 9
+ 2 = 4 2 + 2 4 2
y2 z π 2 AB · CA π CA · AB
2 2
9CA + 9AB
= 4 4
π 2 AB · CA
2
9BC
= 4 4
π 2 AB · CA
 2
3BC
= 2 2
πAB · CA
1
= .
x2
GABARITO MA13 - Avaliação 1 - 2o semestre - 2013

Questão 1. (pontuação: 2)

ABCDE é um pentágono regular e ABF é um triângulo equilátero interior ao pentágono. Calcule os ângulos internos
do triângulo AF C.

Uma solução:

Cada ângulo interno do pentágono regular mede

180o (5 − 2)
= 108o = AB̂C
5
180o − 108o
Sendo AB = BC, então B ÂC = B ĈA = = 36o . Assim, F ÂC = F ÂB − C ÂB = 60o − 36o = 24o .
2
Temos ainda F B̂C = AB̂C − AB̂F = 108o − 60o = 48o .
180o − 48o
Sendo BF = BC, então B F̂ C = B ĈF = = 66o . Assim, AĈF = B ĈF − B ĈA = 66o − 36o = 30o .
2
Finalmente, AF̂ C = AF̂ B + B F̂ C = 60o + 66o = 126o .

Conclusão: os ângulos internos do triângulo AF C medem 24o , 30o e 126o .

Questão 2. (pontuação: 2)

No triângulo ABC tem-se AB = c, BC = a e AC = b. A semirreta AD (D ∈ BC) é bissetriz do ângulo BAC e o


ponto I é o incentro do triângulo.
IA
a) (1,0) Calcule a razão em função dos lados a, b e c.
ID
b+c
b) (1,0) Sendo G o baricentro de ABC mostre que, se IG é paralelo a BC, então a = .
2
Uma solução:
BD AB c
a) Pelo teorema da bissetriz interna, temos = = . Então, devido a uma propriedade útil das proporções,
DC AC b
BD DC a
= = , pois BD + DC = a
c b b+c
ac
Logo BD = .
b+c
Sendo I o incentro então BI é bissetriz do ângulo AB̂C. Pelo mesmo teorema,

IA BA c b+c
= = ac = a
ID BD
b+c

b) Seja M o ponto médio de BC. Como IG é paralelo a BC então

IA b+c GA 2
= = =
ID a GM 1
b+c
(a última igualdade é válida pois G é o baricentro de ABC). Logo, a = , c.q.d.
2

Questão 3. (pontuação: 2)

No triângulo acutângulo ABC tem-se AB = x − 1, BC = x e AC = x + 1.


a) (1,0) Determine todos os valores possı́veis de x para que exista um triângulo nas condições descritas acima.
b) (1,0) Seja D o ponto do lado BC tal que AB = AD. Calcule o comprimento do segmento DC.

Uma solução:
a) Note que AC é o maior lado. Para que ABC seja acutângulo, o maior ângulo deve ser menor do que 90o . Isto
significa que devemos ter AC 2 < AB 2 + BC 2 . De (x + 1)2 < (x − 1)2 + x2 concluı́mos que x > 4.
(O “caso limite” x = 4 é o conhecido triângulo retângulo 3 − 4 − 5)

b) Seja AH perpendicular a BC. Como AB = AD, então H é médio de BD. Sejam AH = h, BH = a e HC = b.


Nos triângulos retângulos AHC e AHB temos:

HC 2 = AC 2 − AH 2 ou seja b2 = (x + 1)2 − h2

HB 2 = AB 2 − AH 2 ou seja a2 = (x − 1)2 − h2
Subtraindo,

b2 − a2 = (x + 1)2 − (x − 1)2 − 4x

ou seja, (b − a)(a + b) = 4x. Como a + b = BC = x então b − a = DC = 4.

Questão 4. (pontuação: 2)

O quadrilátero ABCD está inscrito em uma circunferência. Seja M o ponto médio do arco CD como mostra a figura.
Os segmentos M A e M B cortam o lado CD em P e Q, respectivamente.

a) (1,0) Mostre que o quadrilátero ABQP é inscritı́vel.


b) (1,0) Mostre que os ângulos DÂQ e P B̂C são iguais.

Uma solução:
a) Sejam a, b, c, d e novamente d os arcos DA, AB, BC, CM e M D como na figura acima. Vamos calcular a
soma de dois ângulos opostos do quadrilátero ABQP .

arc(BM ) c+d
P ÂB = M ÂB = =
2 2

arc(DB) + arc(CM ) a+b+d


B Q̂P = B Q̂D = =
2 2
(já que B Q̂P é angulo externo ao triângulo DQM ). Assim

a+b+c+d+d 360o
P ÂB + B Q̂P = = = 180o
2 2
Logo, ABQP é inscritı́vel.

b)
d
DÂQ = DÂP + P ÂQ = DÂM + P ÂQ = + P ÂQ
2
e

d
P B̂C = P B̂Q + QB̂C = P B̂Q + M B̂C = P B̂Q +
2

Como ABQP é inscritı́vel então P ÂQ = P B̂Q. Logo DÂQ = P B̂C, c.q.d.

Questão 5. (pontuação: 2)

Considere um triângulo ABC inscrito em uma circunferência de centro O. Os pontos D e E das retas BC e AC,
respectivamente, são tais que AD é perpendicular a BC e BE é perpendicular a AC. As retas AD e BE cortam-se
em H. Sejam M , N e P os pontos médios dos segmentos AC, AH e AB, respectivamente.

a) (1,0) Mostre que OM N P é um paralelogramo.

b) (1,0) Mostre que em um triângulo qualquer, a distância do ortocentro a um vértice é o dobro da distância do
circuncentro ao lado oposto.

Uma solução:
a) O ponto O é o circuncentro de ABC e, portanto, pertence as mediatrizes dos lados do triângulo. Assim, OM é
perpendicular a AC e OP é perpendicular a AB. Como P e N são médios de AB e AH então P N é paralelo a BH
que, por sua vez, é perpendicular a AC. Logo, P N e OM são paralelos porque são ambos perpendiculares a AC.
Agora, AD e BE são alturas do triângulo ABC e seja CF a terceira altura. Logo, CF passa por H, o ortocentro
do triângulo. Repetindo o argumento, como N e M são pontos médios de AH e AC então M N é paralelo a CH
que, por sua vez, é perpendicular a AB. Logo, M N e OP são paralelos porque são ambos perpendiculares a AB.
Assim, OM N P é um paralelogramo.

b) A distância do circuncentro O ao lado AC é OM . No triângulo AHB, como P e N são médios de AB e AH


respectivamente, então BH é o dobro de P N que, por sua vez, é igual a OM . Assim, BH = 2.OM , como querı́amos
demonstrar.

Obs: Naturalmente que essa propriedade vale qualquer que seja o vértice do triângulo.
GABARITO MA13 Geometria I - Avaliação 2 - 2013/2

Questão 1. (pontuação: 2)

As retas r, s e t são paralelas, como mostra a figura abaixo. A distância entre r e s é igual a 3 e a distância entre
s e t é igual a 1. O triângulo equilátero ABC possui os vértices A, B e C sobre as retas r, s e t, respectivamente.
Determine o lado do triângulo ABC.

Uma solução:
Seja AB = BC = CA = 4a. Sendo D o ponto de interseção da reta s com o lado AC temos, pelo teorema de
Tales, AD = 3a e DC = a .

Vamos calcular a área de ABC de duas formas. √


(4a)2 3 √
a) A área de um triângulo equilátero de lado 4a é S = = 4 3a2 .
4
b) A área de ABC é a soma das áreas dos triângulos BDA e BDC que possuem base comum BD = x e alturas 3
x.3 x.1 √
e 1, respectivamente. Então, S = + = 2x. Assim, 4 3a2 = 2x.
2 2
Que relação há entre a e x?
A relação de Stewart no triângulo ABC com a ceviana AD fornece

16.a2 .3a + 16a2 .a = x2 .4a + a.3a.4a



ou seja, x = a 13. √
√ 2 √ 2 √ 39
Como 4 3a = 2x então 4 3a = 2a 13, ou seja, a = .
√ 6
2 39
Então o lado do triângulo é AB = 4a = .
3

Outra solução
Seja y o lado do triângulo e sejam α e β os ângulos que BA e BC fazem com uma reta perpendicular às paralelas,
respectivamente.

s s
y2 − 9 y2 − 1
r r
3 9 1 1
Como cos α = então sen α = 1 − 2 = 2
e como cos β = então sen β = 1 − 2 = .
y y y y y y2
Como ∠ABC = 60o então α + β = 120o . Assim, cos α.cos β − sen α.sen β = − 21 .
Daı́, fazendo as substituições, temos
p p
3 1 y2 − 9y2 − 1 1
. − . =−
y y y y 2
r √
52 2 39
Após as necessárias manipulações algébricas encontramos y = = .
3 3

Mais uma solução


Primeiro traçamos duas perpendiculares às retas paralelas, passando por B e C, como na figura:
y z

Assim x = y + z e, aplicando três vezes o teorema de Pitágoras, temos




 x=y+z

 l 2 = x2 + 1



 l2 = y 2 + 9

 2
l = z 2 + 16
Apesar de não linear, este sistema pode ser facilmente resolvido. De fato,

p p p
l2 − 1 = l2 − 9 + l2 − 16

Elevando ao quadrado, isolando √ o termo que aparecerá ainda com a raiz quadrada e elevando novamente ao
2 39
quadrado, chegamos à solução l = .
3
Obs: Há várias outras soluções.

Questão 2. (pontuação: 2)

Um poliedro convexo P possui 8 vértices, apenas uma face pentagonal e todas as outras faces triangulares.

(1,0) a) Determine o número de faces triangulares de P .


(1,0) b) Determine o número de diagonais de P .

Uma solução:
a) Como P possui 8 vértices e uma face pentagonal (5 vértices), então há uma face triangular oposta à essa face
pentagonal. Fazendo um desenho e unindo os vértices dessas duas faces formando triângulos, obtemos o poliedro
abaixo.
Como mostra o desenho, há 9 faces triangulares.

Outra forma de chegar à mesma conclusão:


No poliedro P temos uma face pentagonal e x faces triangulares: F3 = x e F5 = 1. Como 2A = 3F3 + 5F5 , temos
2A = 3x + 5, ou seja,

3x + 5
A=
2
3x + 5
A relação A + 2 = F + V fornece + 2 = x + 1 + 8, pois só há faces triangulares e uma pentagonal. A solução
2
dessa equação é x = 9. O poliedro P possui 9 faces triangulares.

b) A face pentagonal possui 5 diagonais. Essas diagonais não são diagonais de P . O poliedro P possui 8 vértices,
10 faces e 16 arestas. O número de diagonais de P é

d = C82 − 16 − 5 = 7

Questão 3. (pontuação: 2)

O quadrado ABCD está contido no plano Π e DE é um segmento perpendicular a Π. Trace os segmentos EA,
EB e EC formando a pirâmide EABCD. Considere AB = 2 e DE = 4.
(0,5) a) Calcule o cosseno do ângulo que a reta BE faz com Π.
(0,5) b) Calcule o cosseno do ângulo que o plano EAB faz com Π.
(1,0) c) Calcule o cosseno do ângulo entre os semiplanos EBA e EBC.

Uma solução:


a) O ângulo que BE faz com o plano da base da pirâmide é o ângulo DB̂E = α. Como AB = 2 então BD = 2 2

e, no triângulo EDB, retângulo em B temos, pelo teorema de Pitágoras, EB = 2 6 . Assim,
√ √
BD 2 2 1 3
cos α = = √ =√ = .
BE 2 6 3 3

b) ED é perpendicular a DA e DA é perpendicular a AB. Então, pelo teorema das três perpendiculares, EA é


perpendicular a AB. Assim, o ângulo que o plano EAB faz com o plano da base da pirâmide é o ângulo DÂE = β.

No triângulo retângulo DAE calculamos AE = 2 5 e, em seguida,

AD 2 5
cos β = = √ = .
AE 2 5 5

c) O plano que contém AC e é perpendicular a BE corta BE em P . O ângulo entre os semiplanos EBA e EBC
é o ângulo AP̂ C = θ.
Sejam P A = P C = x (BDE é o plano mediador de AC). No triângulo EAB o segmento P A é a altura relativa à
hipotenusa.

√ √ 2 5 10
Como, AE.AB = EB.P A, temos 2 5.2 = 2 6.x, ou seja, x = √ . Assim, x2 = .
6 3
No triângulo AP C, usando a lei dos cossenos relativa ao vértice P , temos:

AC 2 = P A2 + P C 2 − 2.P A.P C.cos θ

Então,


(2 2)2 = x2 + x2 − 2.x.x.cos θ ⇒ 8 = 2x2 − 2x2 cos θ ⇒ 4 = x2 − x2 cos θ

10 10 1
Logo 4 = − cos θ ⇒ cos θ = − .
3 3 5
Questão 4. (pontuação: 2)

A figura abaixo mostra duas circunferências de raio 1, tangentes entre si e inscritas em uma semicircunferência de
diâmetro AB.

A B

(0,5) a) Calcule o comprimento do segmento AB.


(1,5) b) Calcule a área da região sombreada.

Uma solução:

Observando a figura a seguir, sejam: O, o centro da semicircunferência, OC o raio da semicircunferência perpen-


dicular a AB, K, o centro da circunferência da direita, KE e KF , raios dessa circunferência perpendiculares a OB
e OC respectivamente, e OD, o raio da semicircunferência que passa por K.

a) OEKF é um quadrado de lado 1. Como D é o ponto de tangência entre a circunferência da direita e a



semicircunferência, então OD = OK + KD = 2 + 1.

Assim, AB = 2( 2 + 1).

b) Seja S a área da região sombreada. Para calcular a área dessa região, transferimos a sua metade da esquerda
da para a região EBD como mostra a figura acima. Assim, S é a área sombreada na figura acima, que é igual à área
do quadrante circular OBC subtraı́da da área do quadrado OEKF e da área do setor KEDF K.

π( 2 + 1)2 3π
S= −1−
4 4

2π − 2
Simplificando, obtemos S = .
2

Questão 5. (pontuação: 2)

Considere uma esfera de centro O e raio 1, e seja AB um diâmetro dessa esfera. Um cone de revolução possui
vértice A e base de centro B e raio 1. A figura abaixo mostra a seção nesses sólidos por um plano que contém a reta
AB.

(1,0) a) Mostre que a circunferência que é a interseção da superfı́cie da esfera com a superfı́cie lateral do cone tem
4
raio igual a .
5
(1,0) b) Calcule o volume da parte comum entre a esfera e o cone (relacionado com a área sombreada da figura).

Obs: Você pode usar o resultado do item a) mesmo que não o tenha demonstrado.

Uma solução:
Como na figura abaixo, seja BD o raio da base do cone e CE o raio da circunferência Γ, intersecção da superfı́cie
da esfera com a superfı́cie lateral do cone.
BD 1
Se ∠DAB = α então tan α = = .
BA 2
EB
O triângulo AEB é retângulo em E. Assim, fazendo EB = b, temos tan α = e, portanto, EA = 2b.
EA
4
O teorema de Pitágoras no triângulo AEB fornece b2 = .
5
O segmento CE = r é altura relativa à hipotenusa do triângulo AEB. Assim, EB.EA = AB.CE, ou seja,
4
b.2b = 2.r e então r = b2 = .
5

b) A parte comum entre a esfera e o cone dado é formada por um cone de altura CA cuja base é a circunferência
Γ, reunido com o segmento esférico com base na circunferência Γ e contendo o ponto B.
Como OE = 1 e CE = 45 , então CO = 35 .
3 8
Assim a altura do cone é H = CA = 5 +1= .
5
2
A altura do segmento esférico é h = 1 − 35 = .
5
O volume da parte comum entre a esfera e o cone dado é

1 2 π.h2
V = π.r .H + (3R − h)
3 3

π 4 2 8 π 2 2 2 π 128 π 52 π 180
V = .( ) . + .( ) .(3.1 − ) = . + . = .
3 5 5 3 5 5 3 125 3 125 3 125

Logo

12π
V =
25
.
AV1 - MA 13 - 2011

Questão 1.
A figura abaixo mostra uma sequência de circunferências de centros C1 , C2 , . . ., Cn com raios r1 , r2 , . . ., rn , respec-
tivamente, todas tangentes às retas s e t, e cada circunferência, a partir da segunda, tangente à anterior.

C1
C2
C3

Considere r1 = a e r2 = b.

(1,0) (a) Calcule r3 em função de a e b.

(1,0) (b) Calcule rn em função de a e b.

UMA SOLUÇÃO

C1
a
b C2
A b x C3
B

(a) Todos os centros estão a igual distância das duas retas, portanto estão na bissetriz das retas s e t. Seja A o ponto
de intersecção entre a paralela à reta t passando por C2 e a perpendicular à reta t passando por C1 , e seja B o ponto
de intersecção entre a paralela à reta t passando por C3 e a perpendicular à reta t passando por C2 . Seja x = r3 .

1
Como os triângulos-retângulos AC1 C2 e BC2 C3 são semelhantes, temos

C1 A C B
= 2 ,
C1 C2 C2 C3
isto é,
a−b b−x
= ,
a+b b+x
b2
o que implica x = a .

(b) A relação obtida


r22
r3 =
r1
pode ser reformulada como
r3 r b
= 2 = ,
r2 r1 a
o que mostra que os três raios formam uma progressão geométrica de razão ba . Como a mesma situação ocorre para
b
quaisquer três circunferências consecutivas, a sequência r1 , r2 , . . ., rn , . . . é uma progressão geométrica de razão a e
termo inicial a. Assim   n −1
b b n −1
rn = a · = n −2 ,
a a
para n = 1, 2, 3, . . ..

2
AV1 - MA 13 - 2011

Questão 2.
Na figura abaixo, a circunferência de centro I é tangente em D ao lado BC do triângulo ABC e é tangente em E e
F aos prolongamentos dos lados AB e AC, respectivamente.

F
C

I
D

A
B E

(1,0) (a) Mostre que AE é igual ao semiperímetro do triângulo ABC.

(1,0) (b) Mostre que o ângulo A b


IB é a metade do ângulo ACB.
b

UMA SOLUÇÃO

(a) Seja 2p o perímetro do triângulo ABC. Tem-se

2p = AB + BC + CA = AB + BD + DC + CA = AB + BE + CF + CA = AE + AF = 2AE .

Logo AE = p.

b =A
(b) No triângulo ABC, sejam B AC b = C.
b e ACB b =A
b O ângulo externo de vértice B é D BE b + C. IB = θ.
b Seja A b
Como AI e BI são bissetrizes dos ângulos C AB
b e D BE
b então, no triângulo ABI, o ângulo externo I BE
b é tal que

b+C
A b D BE
b A
b
= = I BE
b = I AB
b + Ab
IB = +θ.
2 2 2
Logo
C
b
θ= .
2

3
AV1 - MA 13 - 2011

Questão 3.

(2,0) Dado um paralelogramo ABCD construa no seu exterior os triângulos equiláteros BCE e CDF. Mostre que o
triângulo AEF é equilátero.

UMA SOLUÇÃO

A D

α
B
C
F

Primeiro, vemos que BA = DF = CF. A segunda igualdade é consequência de CDF ser equilátero, enquanto a
primeira segue de que AB = CD (pois ABCD é paralelogramo) e CD = DF (pois CDF é equilátero).
Depois, vemos que AD = BE = EC. A segunda desigualdade segue de BCE ser equilátero. A primeira segue de
que AD = BC (pois ABDC é paralelogramo) e BC = BE (pois BCE é equilátero).
Finalmente, vamos mostrar que os ângulos A BE, b ECF
b e A DFb são iguais. Para isso vamos mostrar que todos
são iguais a α + 60o , em que α é o ângulo A BC.
b De fato, isso é evidente para A BE,
b pois BCE equilátero implica
b = 60o . O mesmo para A DF,
C BE b = 60o
b = α (ângulos opostos do paralelogramos são iguais) e C DF
b pois A DC
(CDF é equilátero). Finalmente, em torno do ponto C tem-se

b + D CF
BCD b + F CE b = 360o ,
b + ECE

logo
(180o − α) + 60o + ECF
b + 60o = 360o

b = α + 60o , como queríamos demonstrar.


e, portanto, ECF
Portanto os triângulos ABE, FCE e FDA são congruentes, de onde concluímos que AE = EF = AF, isto é, AEF é
equilátero.

4
AV1 - MA 13 - 2011

Questão 4.

(2,0) No triângulo ABC, Bb = 68o e C


b = 40o , AD e BE são alturas, M é médio de BC e N é médio de AC. Calcule os
ângulos D N
b M e E DN.
b

UMA SOLUÇÃO

A
E

B D M C

(A figura não foi desenhada com os ângulos prescritos no enunciado)

b = 180o − 68o − 40o = 72o . Segundo, como N é o ponto médio de AC, então é equidistante de
(a) Primeiro, B AC
A e D. Logo AND é isósceles e ND = N A. Pela mesma razão N A = NC, de onde resulta que NDC é isósceles.
b = 40o e que D NC
b = ACB
Disso resulta que N DC b = 180o − 40o − 40o = 100o . Terceiro, MN é paralelo a BA, logo
MNC é semelhante a BAC e, por conseguinte, M NC b isto é, 72o . Portanto, D N
b é igual a B AC, b − M NC
b M = D NC b =
100o − 72o = 28o .

b = 180o − D NC
(b) ADN é isósceles e A ND b = 80o , logo A DN
b = 50o .
b = 90o = B EA,
Como B DA b então E e D pertencem à circunferência cujo diâmetro é AB. Logo, os ângulos A BE
b e
A DE
b inscritos nessa circunferência são iguais. Então A DE b = 90o − 72o = 18o .
b = A BE
b = A DN
Portanto E DN b = 50o − 18o = 32o .
b − A DE

5
AV1 - MA 13 - 2011

Questão 5.

(2,0) O triângulo equilátero ABC está inscrito em uma circunferência e P é um ponto qualquer do menor arco BC.
Prove que PA = PB + PC (isto é, que a distância de P ao ponto A é igual à soma das distâncias de P aos pontos
B e C).
Sugestão: Considere um ponto D sobre PA tal que PD = PB.

UMA SOLUÇÃO

B C

Seja D o ponto do segmento PA tal que PD = PB. Precisamos mostrar que AD = PC.
Como o arco AB mede 120o , então B PA
b = 60o . Então B PD
b = 60o (é o mesmo ângulo) e, como PB = PD, então
b = 60o e, por conseguinte,
PBD é equilátero, resultando que BD = PB. Também por PBD ser equilátero tem-se B DP
b = 120o .
B DA
b = 240o = 120o , logo B PC
Como o arco BAC mede 240o , então B PC b = B DA.
b Juntando essa informação com a
2
igualdade B AP = BCP, que é evidente da simetria da construção, concluímos que A BD
b b b = P BC.
b
Por LAL os triângulos ABD e CBP são congruentes, resultando que AD = PC, como queríamos demonstrar.

6
MA13 – Geometria I – Avaliação 2 – 2011
Gabarito

Questão 1
(2,0) A figura abaixo mostra um triângulo equilátero e suas circunferências inscrita e
circunscrita. A circunferência menor tem raio 1.
Calcule a área da região sombreada.

Uma solução:

X
O
Y
B M C

Seja O, o centro do triângulo equilátero ABC e seja M o ponto médio do lado BC como na
figura acima. Pela propriedade do baricentro do triângulo, OA 2 OM e como OM 1 temos
OA 2 .
A região cuja área se pede é formada por duas partes justapostas X e Y como mostra a figura.
Observando que 3 X 3Y é a área da coroa circular formada pelas duas circunferências temos
3( X Y ) 22 12 3 .
Logo, X Y .
Questão 2
O poliedro P que inspirou a bola da Copa de 70 é formado por faces
pentagonais e hexagonais, e é construído da seguinte forma:
•Considere um icosaedro regular de aresta a (Fig. 1 abaixo).
•A partir de um vértice e sobre cada uma das 5 arestas que concorrem nesse
a
vértice, assinale os pontos que estão a uma distância de desse vértice. Esses
3
5 pontos formam um pentágono regular (Fig. 2).
•Retirando a pirâmide de base pentagonal que ficou formada obtemos a Fig. 3.
•Repetindo a mesma operação para todos os vértices do icosaedro obtém-se o poliedro P.

Fig. 1 Fig. 2 Fig. 3

(0,5) (a) Determine quantas são as faces pentagonais e quantas são as faces hexagonais de P.

(0,7) (b) Determine os números de arestas, faces e vértices de P.

(0,8) (c) Sabendo que uma diagonal de um poliedro é todo segmento que une dois vértices que
não estão na mesma face, determine o número de diagonais de P.

Uma solução:

(a) Cada face pentagonal de P apareceu onde havia um vértice do icosaedro. Como o icosaedro
tem 12 vértices então P tem 12 faces pentagonais. Cada face (triangular) do icosaedro deu
origem a uma face hexagonal de P. Como o icosaedro tem 20 faces triangulares então P tem 20
faces hexagonais.

(b) Do item anterior temos F5 12 e F6 20


O número total de faces de P é F F5 F6 12 20 32 .
Contando as arestas temos: 2 A 5 F5 6 F6 5 12 6 20 180 , ou seja, A 90 .
Como P é convexo então vale a relação de Euler V A F 2 . Portanto, V 60 .

(c) Seja d n o número de diagonais de um polígono de n lados.


O número de diagonais de um pentágono é d 5 5 e o de um hexágono é d 6 9.
A soma dos números de diagonais de todas as faces é S F5 d 5 F6 d 6 12 5 20 240 .
Vamos agora construir todos os segmentos cujas extremidades são os V vértices do poliedro P.
A quantidade de diagonais de P é D CV2 A S .
60 59
Assim, D C602 90 240 90 240 1170 330 1440.
2
Questão 3

Definição: Dado um segmento AB, o plano mediador desse segmento é o plano perpendicular a
AB que contém o seu ponto médio.

1ª Parte
(2,0) Prove que um ponto P equidista de dois pontos A e B se, e somente se, pertence ao plano
mediador de AB.

Uma solução:

Seja M o ponto médio de AB e seja Π o plano mediador de AB.

A
(a) Suponha que P pertença a Π. Se P coincide com M então
Π equidista de A e B. Se não, como AB é perpendicular a Π então AB
P M é perpendicular a MP. Como M é médio de AB então os triângulos
retângulos MPA e MPB são congruentes.
Logo, PA PB , ou seja, P equidista de A e B.
B

A (b) Suponha que P não pertença a Π. Imaginemos, por exemplo e


sem perda de generalidade, os pontos P e A no mesmo semiespaço
P Π determinado por Π. Como B está no semiespaço oposto a reta PQ
corta Π em um ponto Q. Como Q então, pela parte a),
Q
QA QB .
No triângulo PAQ tem-se: PA PQ QA PQ QB PB .
B
Assim, P não equidista de A e B.
2ª Parte
A figura abaixo mostra o cubo ABCD-EFGH de aresta a. H
Sejam M, N, P, Q, R e S os pontos médios das arestas G
AB, BF, FG, GH, HD e DA. E
F
(0,5) (a) Mostre que esses seis pontos são coplanares.
Sugestão: Mostre que qualquer um deles pertence ao plano
mediador da diagonal EC do cubo (a propriedade enunciada na
primeira parte da questão pode ser utilizada mesmo que você não a D
tenha demonstrado). C

A
(0,5) (b) Mostre que o hexágono MNPQRS é regular. B

(1,0) (c) Calcule o volume da pirâmide de vértice E e base MNPQRS.

Uma solução:
(a) Tomemos o ponto M, médio da aresta AB. Os
triângulos AME e BMC são congruentes, pois AM BM ,
E AE BC e MAE MBC 900
Logo, ME MC e, portanto, M pertence ao plano
mediador da diagonal EC.
Analogamente, cada um dos outros pontos: N, P, Q, R e S
C também estão nesse mesmo plano.
A M B
(b) Cada lado do hexágono é a metade da diagonal de
H Q BG a 2
G uma face. Por exemplo, NP .
2 2
E P Seja O, o centro do cubo. Todos os vértices do
R F hexágono possuem mesma distância ao ponto O. A
O
distância do centro do cubo a qualquer aresta é a
a 2
metade da diagonal de uma face, ou seja, .
N 2
S Portanto, cada um dos triângulos MON, NOP, ...,
C
SOM é equilátero e o hexágono é regular.
A
M B
a2 3 3 3a 2
(c) A área do hexágono é 6 .
4 2
a 3
Como a altura da pirâmide é a metade da diagonal do cubo temos OE .
2
1 3 3a 2 a 3 3a 3
O volume da pirâmide é: V .
3 2 2 8
H
3ª Parte G
A figura abaixo mostra o cubo ABCD-EFGH de aresta a.
E
(1,0) (a) Mostre que as retas DB e EC são ortogonais. F

(1,0) (b) Calcule o comprimento da perpendicular


comum entre DB e EC. D
C

A
B

Uma solução:
H
G

Y
D
C
X
A
B

(a) Seja Π o plano diagonal AEGC.


Como AE é perpendicular ao plano ABCD então AE é ortogonal a BD. Mas AC é perpendicular
a BD (pois as diagonais de um quadrado são perpendiculares. Como BD é ortogonal a AE e AC
então BD é perpendicular a Π.
Como EC está contida em Π então BD é ortogonal a EC.

(b) Seja X o ponto onde BD fura o plano Π. O ponto X é o centro da face ABCD.
Sobre o plano Π tracemos XY perpendicular a EC.
Lembrando que BD é perpendicular a Π então BD é perpendicular a XY. Assim, XY é a
perpendicular comum entre BD e EC.

Os triângulos retângulos CYX e CAE são semelhantes. Logo,

XY CX XY a 2 2 a 6
→ → XY
AE CE a a 3 6
MA13 – Geometria I – Avaliação 3 – 2011

Questão 1
Considere um quadrado ABCD de lado a e seja E o ponto do lado CD tal que
AE BC CE .
(1,0) (a) Calcule o comprimento de CE.
(1,0) (b) Calcule o seno do ângulo CAˆ E .

Questão 2
Um trapézio ABCD tem altura h e bases AB a e CD b . Seja F o ponto de
interseção das diagonais.
(1,0) (a) Calcule as distâncias de F às duas bases.
(1,0) (b) Calcule as áreas dos triângulos ADF e BCF.

Questão 3
Seja ABC um triângulo qualquer. Desenhe exteriormente a ABC os triângulos
equiláteros ABD e ACE.
(1,0) (a) Mostre que DC = BE. Sugestão: use congruência de triângulos.
(0,5) (b) Sendo F o ponto de interseção de DC e BE, mostre que o quadrilátero ADBF
é inscritível.
(0,5) (c) Mostre que AFˆ B BFˆC CFˆA 1200 .

Questão 4
Seja um plano horizontal. A reta r é perpendicular a e seja A o ponto de
interseção de r e . A reta s está contida em e não passa por A. O ponto B da reta s
é tal que AB é perpendicular à reta s. Seja M um ponto de r e N um ponto de s.
Dados: AM a , BN b, AB c .
(0,5) (a) Faça um desenho da situação descrita no enunciado.
(0,5) (b) Calcule a distância entre os pontos M e N.
(0,5) (c) Calcule a tangente do ângulo que a reta MN faz com o plano .
(0,5) (d) Calcule a tangente do ângulo entre as retas AB e MN.

Questão 5
As bases de um tronco de pirâmide regular são quadrados de lados 12 e 4. Sabe-se
que a área lateral é igual à soma das áreas das bases.
(1,0) (a) Calcule a altura do tronco.
(1,0) (b) Calcule o volume do tronco.
MA13 – Geometria I – Avaliação 3 – 2011
Gabarito

Questão 1 – Solução
D E C
(a) Seja CE x . Assim AE a x .
Traçando EF perpendicular a AB temos no triângulo AEF:
a
(a x) 2 (a x) 2 a2 o que dá x . a
4

(b) Seja AEˆ C .


a a 5a
Como CE e AE a temos, pela lei dos senos, A F B
4 4 4
a4 5a 4 2
o que dá sin .
sin 2 2 10

Questão 2 – Solução
D b C
(a) Sejam x e y as distâncias de F às bases AB e y
CD, respectivamente. Como os triângulos FAB e F
FCD são semelhantes, temos: h
a b a b x
x y h
ah bh A a B
Assim, x e y .
a b a b

(b) Os triângulos ADB e ACB têm mesma área porque possuem mesma base e mesma
altura. Os triângulos ADF e BCF têm mesma área porque
[ADF] = [ADB] – [AFB] = [ACB] – [AFB] = [BCF]

ah ax a ah abh
[ ADF ] [ BCF ] h
2 2 2 a b 2(a b)
Questão 3 – Solução

(a) Temos AD AB , AC AE e DAˆ C BAˆ E Aˆ 600 . Portanto, os triângulos


ADC e ABE são congruentes e DC = BE.

ˆF
(b) Pela congruência anterior, AD ABˆ F . Portanto D está na circunferência que
passa por A, B e F.

(c) Como ADBF é inscritível, seus ângulos são suplementares. Então


AFˆB 1800 AD ˆ B 1800 600 1200 . Analogamente, AECF é inscritível e
CFˆA 1200 . Consequentemente, BFˆC 1200 .

Questão 4 – Solução

(a) r

A
c P

B
b N s

b) No triângulo ABN, retângulo em B, AN 2 b 2 c 2 .


No triângulo MAN, retângulo em A, MN 2 a 2 AN 2 a2 b2 c 2 .
Então MN a2 b2 c2 .

AM a
c) O ângulo que MN faz com é MNˆ A . Assim, tan .
AN b 2
c2

d) Construa o retângulo ABNP.


AM é ortogonal a NP e AP é perpendicular a NP. Portanto, NP é perpendicular ao
plano AMP e, consequentemente, o ângulo NPA é reto.
O ângulo entre MN e BA é o ângulo entre MN e NP, MNˆ P .
PM a2 b2
Assim, tan .
NP c
Questão 5 – Solução

(a)
V
Sejam O e O os centros
das duas bases (maior e
menor) como mostra a O' N
figura acima. 4
Na reta OO está o h x
vértice V da pirâmide que
dou origem ao tronco.
A altura do tronco é
O P M
OO h .
Cada face lateral do 12
tronco é um trapézio
isósceles, e a altura de
um dos trapézios é o segmento MN que une os pontos médios das duas bases. Seja
MN x .
A área lateral do tronco é a soma das áreas dos quatro trapézios. Então,

(12 4) x
4 122 42
2

Isto dá x 5 . Trace agora NP perpendicular à OM como na figura acima. Temos


O O NP h , ON OP 2 , OM 6 e, consequentemente, PN 4 . No triângulo
PMN retângulo em P temos h 3 .

(b) Seja VO y.
y 4 3
Utilizando a semelhança entre as duas pirâmides temos o que dá y .
y 3 12 2
3
9
A altura da pirâmide grande é OV 3 e o seu volume é
2
2
1 9
V1 122 216 .
3 2
1 2 3
O volume da pirâmide pequena é V2 4 8.
3 2
O volume do tronco é a diferença: V 216 8 208 unidades de volume.

Obs:
Pode-se também aplicar a fórmula do volume do tronco de pirâmide:
h
V ( S1 S 2 S1 S 2 ) onde S1 e S 2 são as áreas das duas bases e h é a altura do
3
tronco. Assim,
3 2
V (12 4 2 122 4 2 ) 144 16 48 208 .
3
MA13 Geometria I Avaliação 1 2012

SOLUÇÕES

Questão 1. (pontuação: 2)
O ponto D pertence ao lado AC do triângulo ABC. Sabe-se que AB = BC = CD e que o ângulo ABD mede 21o .
Determine a medida do ângulo ABC.

21o

α α
A D C

Uma solução:
Como AB = BC, seja α = ∠BAC = ∠BCA. O ângulo BDC é externo do triângulo ABD. Então, ∠BDC =
21o + α = ∠DBC, pois BC = CD. No triângulo BDC temos 21o + α + 21o + α + α = 180o , ou seja, α = 46o . O
ângulo ABC mede 21o + 21o + α = 42o + 46o = 88o .

Questão 2. (pontuação: 2)
Quadrados foram construı́dos sobre os lados de um paralelogramo como mostra a figura abaixo. Mostre que os
centros desses quatro quadrados são vértices de outro quadrado.

H x D
y C
x β
α
A B F

E
Uma solução:

No paralelogramo ABCD os quadrados construı́dos sobre os lados AB, BC, CD e DA têm centros E, F , G e H,
respectivamente.
Os triângulos AEB, BF C, CGD e DHA são retângulos e isósceles. O primeiro e o terceiro são congruentes e o
segundo e o quarto são também congruentes.
Sejam ∠BAD = α e ∠ADC = β dois ângulos internos vizinhos do paralelogramo. Sabemos que α + β = 180o .
Observemos que ∠HAE = 45o + α + 45o = 90o + α e que ∠HDG = 360o − 45o − 45o − β = 270o − (180o − α) =
90o + α = ∠HAE.
Reunindo as informações anteriores concluı́mos que os triângulos HAE, HDG, F CG e F BE são todos congruentes
e, portanto, EH = HG = GF = F E e o quadrilátero EF GH possui os quatro lados iguais.
Da congruência dos triângulos HAE e HDG temos ∠AHE = ∠DHG = x e seja ∠EHD = y. Por um lado,
∠AHE + ∠EHD = x + y = 90o , pois o ângulo AHD é reto.
Por outro lado, ∠EHG = ∠DHG + ∠EHD = x + y = 90o .
Assim, o quadrilátero EF GH possui os quatro lados iguais e um ângulo reto. Logo, é um quadrado.

Questão 3. (pontuação: 2)
No triângulo ABC de lados AB = 8, BC = 7 e AC = 9, os pontos M e N dos lados AB e AC, respectivamente,
são tais que o segmento M N é tangente à circunferência inscrita no triângulo ABC. Mostre que o perı́metro do
triângulo AM N é constante e calcule seu valor.

Uma solução:

z
x
y N
M R
P

B Q C

Sejam AM = x, M N = y e N A = z os lados do triângulo AM N . Temos M B = 8 − x e CN = 9 − z . Como o


quadrilátero BCN M é circunscritı́vel temos, pelo teorema de Pitot (Unidade 7, Teorema 4), BC + M N = M B + N C
ou seja, 7+y = 8−x+9−z. Logo x+y +z = 10. Portanto o perı́metro do triângulo AM N é igual a 10, independente
da posição do segmento M N .

Outra solução:
A circunferência inscrita em ABC é uma circunferência exiscrita ao triângulo AM N . Sabemos que o semiperı́metro
do triângulo AM N é o segmento AP que é constante, ou seja, não depende da posição do segmento M N (Unidade
7, Proposição 22). Fazendo AP = AR = a, BP = BQ = b e CQ = CR = c, temos as equações:

 a+b=8


b+c=7


 c+a=9

Resolvendo, encontramos a = 5 que é o semiperı́metro do triângulo AM N . Logo, o perı́metro de AM N é 10.

Questão 4. (pontuação: 2)
No trapézio ABCD os ângulos A e D são retos, AB = 12, CD = 4 e AD = 10. O ponto E pertence ao lado AD e o
ponto F pertence ao lado BC. Sabe-se que as retas EF e AB são paralelas e que o segmento EF fica dividido em
três partes iguais pelas diagonais do trapézio. Calcule a distância entre as retas AB e EF .

Uma solução:
O problema tem duas soluções pois há duas possibilidades: quando EF está abaixo do encontro das diagonais do
trapézio e quando EF está acima do encontro das diagonais do trapézio. Qualquer uma das soluções está igualmente
correta.

Primeira situação:

D 4 C

10 – x

P Q
E F
m m m
10

12 B
A
Na figura acima, seja AE = x. Então, ED = 10 − x. Como as diagonais dividem EF em três partes iguais sejam
m x 2x
EP = P Q = QF = m. Da semelhança dos triângulos AEP e ADC temos: 4 = 10 ⇒ m= 5 . Da semelhança
2m 10−x 3(10−x)
dos triângulos DEQ e DAB temos: 12 = 10 ⇒ m= 5 .
Igualando temos 2x = 3(10 − x), o que dá x = 6.

Segunda situação:

D 4 C
P Q
E m m m F

x
10

A 12 B

Na figura acima, seja AE = x. Então, ED = 10 − x. Como as diagonais dividem EF em três partes iguais sejam
2m 4 x
EP = P Q = QF = m. Da semelhança dos triângulos QEA e CDA temos: x = 10 ⇒ m= 5. Da semelhança
10−x 10 12(10−x)
dos triângulos DEP e DAB temos: m = 12 ⇒ m= 10 .
60
Igualando obtemos x = 7 .

Questão 5. (pontuação: 2)
A figura abaixo mostra o triângulo acutângulo ABC inscrito na circunferência de centro O. A reta BD é perpendicular
em D a AC e encontra a circunferência em M . A reta CE é perpendicular em E a AB e encontra a circunferência
em N . As alturas BD e CE intersectam-se em H, ortocentro do triângulo.
a) Mostre que HD = DM .
b) Mostre que M N é perpendicular a OA.
Uma solução:

M
A

N E
β
β O
H α
α
α
B C

a) Considerando a figura acima, sejam ∠DCH = α e ∠DHC = β. Como o ângulo HDC é reto então α e β
são complementares. Temos ∠EHB = β (oposto pelo vértice de DHC) e ∠HBE = α pois o ângulo BEH é reto.
Escrevemos ∠ABM = ∠HBE = α. Como os ângulos inscritos ABM e ACM subtendem o mesmo arco AM , então
são iguais, ou seja, ∠ACM = α.
Os triângulos retângulos CDH e CDM são congruentes. Assim HD = DM , como querı́amos demonstrar.

b)

A M

N
O
α
α
C

Os arcos AM e AN são iguais porque ∠ACM = ∠ACN = α. Como arcos iguais subtendem cordas iguais o ponto
A equidista dos pontos M e N . Entretanto o ponto O, centro da circunferência também equidista de M e N . Assim,
A e O são pontos da mediatriz do segmento M N o que significa dizer que a reta AO é a mediatriz do segmento M N .
Logo, OA é perpendicular a M N .
MESTRADO PROFISSIONAL EM MATEMÁTICA EM REDE NACIONAL

Avaliação 2 - MA13 - 2015 - Gabarito

Questão 01 [ 2,00 pts ]


Em um ponto da borda de um pasto circular, amarra-se um bode, com uma corda cujo comprimento é 3 vezes
o raio do pasto. O bode pode comer todo o pasto que conseguir alcançar, podendo deslocar-se até a corda estar
totalmente esticada. Determine a fração máxima do pasto que o bode pode comer.

Solução

Sejam O o centro do pasto, R a medida de seu raio e P o ponto onde a corda, de comprimento R 3, está presa.
A área S do pasto que o bode pode comer pode ser decomposto em uma área S1 mais duas áreas idênticas S2 e S3 , conforme
a figura abaixo.


Como OP = OA = R e AP = R 3, temos

 √ 2  
R 3 = R2 + R2 − 2 · R · R · cos P ÔA

logo   1
cos P ÔA = − ,
2
o que nos dá P ÔA = 120◦ . Com isso, como o triângulo P OA é isósceles de vértice O, temos OP̂ A = 21 (180◦ − 120◦ ) = 30◦ .

Teremos então AP̂ B = 60◦ , logo a área do setor circular S1 , de raio R 3 e ângulo AP̂ B = 60◦ é dada por
60◦  √ 2 3πR2 πR2
S1 = ◦
·π R 3 = = .
360 6 2
A área S2 , por sua vez, corresponde à área do setor circular de raio R e ângulo 120◦ , subtraindo a área do triângulo AOP .
Assim,

120◦ 2 R · R · sen 120◦ πR2 R2 3
S2 = · πR − = − .
360◦ 2 3 4
Como S3 = S2 , temos
√ 
πR2 πR2 R2 3

S = S1 + S2 + S3 = S1 + 2 S2 = +2 −
2 3 4
logo √
7π − 3 3
S= · R2 .
6

Questão 02 [ 2,00 pts ]

A figura mostra uma plataforma de base quadrada ABCD e uma torre central em forma de pirâmide quadrangular
de base P QRS e vértice H, cujo volume é 576 m3 . As diagonais P R e QS estão contidas, respectivamente, nas
√ √
diagonais AC e BD. As arestas das bases da plataforma e da pirâmide medem, respectivamente, 19 2 m e 6 2 m.
Para aumentar a segurança, quatro cabos de aço serão presos e esticados entre cada ponto médio das arestas laterais
da pirâmide e o vértice mais próximo da base da plataforma.

(a) Sendo M o ponto médio da aresta HQ e N a projeção de M na base da pirâmide, determine a medida do
segmento M N .

(b) Determine o comprimento do cabo de aço BM .

Solução
(a) Sendo V o volume da pirâmide e O o centro do quadrado P QRS, temos:
1 2 1  √ 2
V = · P Q · OH ∴ 576 = · 6 2 · OH ∴ OH = 24m.
3 3
Como M é ponto médio de QH, então M N é base média do triângulo OQH e, por isso:

OH
MN = ∴ M N = 12m.
2
(b) Aplicando o Teorema de Pitágoras no triângulo P QS, retângulo em P , temos:

2 2 2
 √ 2  √ 2 QS
QS = P Q + P S = 6 2 + 6 2 ∴ QS = 12 ∴ OQ = = 6m.
2
Como o segmento M N é base média do triângulo OQH, então N é ponto médio de OQ, sendo:

OQ
ON = = 3m.
2
Aplicando o Teorema de Pitágoras no triângulo ABD, retângulo em A, temos:

2 2 2
 √ 2  √ 2 BD
BD = AB + AD = 19 2 + 19 2 ∴ BD = 38 ∴ OB = = 19m.
2
Assim segue que:
BN = OB − ON = 19 − 3 ∴ BN = 16m.

Aplicando o Teorema de Pitágoras no triângulo M N B, retângulo em N , temos:


2 2 2
BM = M N + BN = 122 + 162 ∴ BM = 20m.

Questão 03 [ 2,00 pts ]

(a) Prove que todo trapézio inscritı́vel é isósceles.

(b) Um trapézio ABCD, de bases AB e CD, é inscritı́vel e circunscritı́vel. Sabendo que AB > CD, prove que
AB > BC.

Solução
(a) Seja ABCD um trapézio inscritı́vel, com bases AB e CD.

Como o trapézio é inscritı́vel, temos que  + Ĉ = 180◦ . Porém, como AB e CD são paralelos, temos B̂ + Ĉ = 180◦ . As duas
igualdades nos dão, juntas, que  = B̂, o que implica que o trapézio é isósceles.

(b) Como ABCD é inscritı́vel, pelo item (a) ele será isósceles, logo BC = AD.

Por outro lado, o trapézio é circunscritı́vel, logo

AB + CD = BC + AD.

Mas, como BC = AD, temos


AB + CD = 2BC.

E, como AB > CD, temos


2AB = AB + AB > AB + CD = 2BC,

logo
AB > BC.

Questão 04 [ 2,00 pts ]

Seja ABCD um losango de diagonais de medidas AC = 2a e BD = 2b. Dos pontos A e C, traçamos os segmentos
AE e CF , de medidas AE = x e CF = y, perpendiculares ao plano que contém o losango e de um mesmo lado deste
plano. Calcule o volume do sólido ABCDEF .
Solução

Como pode ser visto na figura, o sólido ABCDEF é formado pela união de duas pirâmides:
• a pirâmide cuja base é o trapézio ACF E e cuja altura é P B e
• a pirâmide cuja base é o trapézio ACF E e cuja altura é P D,
sendo P o ponto de encontro das diagonais do losango ABCD.
Como P B = P D = b, o volume de cada uma dessas pirâmides é dado por
1 1 x + y  ab(x + y)
V1 = · ÁreaACF E · b = · · 2a · b = .
3 3 2 3
Assim, o volume do sólido ABCDEF é
2ab(x + y)
VABCDEF = 2 · V1 = .
3

Questão 05 [ 2,00 pts ]

(a) Usando apenas a relação fundamental da trigonometria e as fórmulas de adição de arcos e de arcos duplos prove
que:

cos(3x) = 4 cos3 (x) − 3 cos(x).


π
(b) Resolva a equação 4 cos3 (x) − 3 cos(x) = cos .
5

Solução
(a) Aplicando a fórmula de adição de arcos temos:

cos(3x) = cos(2x + x) = cos(2x) cos(x) − sen(2x)sen(x).

Pelas fórmulas de arcos duplos segue que:

cos2 (x) − sen2 (x) cos(x) − [2sen(x) cos(x)] sen(x)


 
cos(3x) =
= cos3 (x) − 3sen2 (x) cos(x)
cos3 (x) − 3 1 − cos2 (x) cos(x)
 
=
= 4 cos3 (x) − 3 cos(x).

(b) Por (a) segue que 4 cos3 (x) − 3 cos(x) = cos(3x).

π
Logo cos(3x) = cos .
5

π π 2
Portanto 3x = ± + 2kπ, k ∈ Z, ou seja, x = ± + kπ, k ∈ Z.
5 15 3
MESTRADO PROFISSIONAL EM MATEMÁTICA EM REDE NACIONAL

Avaliação 1 - MA13 - 2015 - Gabarito

Questão 01 [ 2,00 pts ]

Em um quadrilátero convexo ABCD, prove que

AB + CD
MN 6 ,
2
onde M e N são os pontos médios dos lados BC e AD, respectivamente.

Sugestão: Utilize o ponto médio da diagonal AC.

Solução

AB CD
Tomando P como ponto médio de AC, temos que M P = e PN = . Para o ponto P há duas situações possı́veis :
2 2
(I) M , N e P não estão alinhados. Da existência do triângulo M N P , teremos:

MN < MP + P N.

(II) M , N e P estão alinhados, ou seja,


MN = MP + P N.

De (I) e (II) :
AB + CD
MN 6 .
2
Questão 02 [ 2,00 pts ]

Na figura, M é ponto médio de AB.

Descreva a construção com régua e compasso da circunferência C, tangente à reta r e ao segmento AB, e tal que M
seja o ponto de tangência de C com AB.

Solução

Vamos imaginar a circunferência construı́da, para buscar um ponto chave da construção.

Como a circunferência é tangente ao segmento AB em seu ponto médio M , o centro O estará sobre a mediatriz m de AB. E,
↔ ↔
como a circunferência é tangente tanto a AB quanto à reta r, seu centro estará sobre a bissetriz b entre r e AB.
Assim, basta construir m e b.
1
Para construir m, tomamos o compasso e, com uma mesma abertura maior que 2
AB, traçamos duas circunferências, uma
de centro A e outra de centro B. Estas duas circunferências irão se intersectar em dois pontos, X e Y . Construı́mos então

m =XY .

Para construir m, prolongamos AB até C ∈AB ∩r. Com um raio qualquer, construı́mos um cı́rculo de centro C, que
−→ −→
intersectará CA em um ponto W . Esta circunferência intersectará r dois pontos, escolhemos um deles, Z, de forma que CZ
determine o menor ângulo Z ĈA, como na figura. Com abertura maior que 12 W Z, construı́mos duas circunferências, de centros

W e Z respectivamente, que se intersectam em dois pontos. Sendo T um destes pontos, b =CT .
Assim, construı́das as retas m e b, obtemos O ∈ m ∩ b. Para construir a circunferência pedida, basta fazer O como centro e
tomar OM como abertura do compasso.

Questão 03 [ 2,00 pts ]

−−→ −−→
Em um triângulo ABC de incentro I, sejam E e F pontos sobre AC tais que BE é bissetriz de ∠ABC e BF é
bissetriz de ∠EBC. Se ∠BAC mede 60◦ e F I é paralelo a BC, determine as medidas dos ângulos ∠ABC e ∠ACB.
Dica: Procure uma relação entre os triângulos ABI e AF I.

Solução

Seja ∠BAC = 4θ, teremos então que ∠ABE = ∠EIF = 2θ e que o triângulo BIF é isósceles.
Sejam M e N os pés das perpendiculares traçadas de I sobre AB e AC, respectivamente.
Da congruência dos triângulos BIM e N IF , temos que ∠EF I = 2θ e consequentemente no triângulo ABC obtemos 4θ +
2θ + 60◦ = 180◦ , ou seja, θ = 20◦ .
Logo os ângulos pedidos são 80◦ e 40◦ .
Questão 04 [ 2,00 pts ]

Em um trapézio de bases AB e CD, com AB > CD, traça-se por B uma reta paralela à diagonal AC que encontra
o prolongamento de AD em E. Sendo P o ponto de encontro dos prolongamentos dos lados AD e BC, determine
P A em função apenas de P D e P E.

Solução

Sejam DP = a, P E = b, DC = x, AB = y, AC = n e EB = m. Queremos determinar AP = k como função de a e b.


Das semelhanças dos triângulos AP C e P EB, ADC e ABE, ABP e P DC, temos que
b m y m y k
= , = , = .
k n x n x a

Das igualdades acima encontramos

b k √
= , donde k = ab.
k a
Questão 05 [ 2,00 pts ]

2 2 2
Seja P um ponto sobre a diagonal AC do quadrado ABCD. Prove que P A , P B , P C estão, nesta ordem, em
progressão aritmética.

Solução

Sejam Q, R e S os pés das perpendiculares traçadas de P sobre os lados AB, AD e CD, respectivamente. Como P está sobre
a diagonal AC, temos que P Q = AQ = x e P S = SC = QB = L − x, onde L é a medida do lado do quadrado.

Dos triângulos retângulos P QA, P QB e P SC teremos:


2
P A = x2 + x2 = 2x2 ,
2
P B = x2 + (L − x)2 ,
2
P C = 2(L − x)2 .

Como
2 2
P C − P B = 2(L − x)2 − x2 + (L − x)2 = (L − x)2 − x2 ,


temos
2 2 2 2
P B − P A = x2 + (L − x)2 − 2x2 = (L − x)2 − x2 = P C − P B .
2 2 2
Com isso, P A , P B e P C estão em uma PA.
MESTRADO PROFISSIONAL EM MATEMÁTICA EM REDE NACIONAL

Avaliação 2 - MA13 - 2015 - Gabarito

Questão 01 [ 2,00 ]

Considere um cilindro sólido de altura 2R, cujas bases são dois cı́rculos de raio R, do qual são retirados dois cones
sólidos de altura R e que têm por base as bases do cilindro, formando-se assim um sólido S. Considere ainda uma esfera

de raio R, e que, assim como o sólido S, está sobre um plano.

(a) Prove que, intersectando a esfera e o sólido S por um plano paralelo ao plano que apoia estes sólidos, como na
figura, obtém-se seções com mesma área.

(b) Supondo conhecidas as expressões do volume do cone e do cilindro, prove que o volume de uma esfera de raio
4
R é dado por πR3 .
3

Solução

(a) Vamos denotar por x a distância do plano ao vértice V dos cones e ao centro O da esfera. Suponhamos inicialmente o
plano abaixo de V e O.

Como na figura, os triângulos V C1 P1 e V C2 P2 são semelhantes, logo

C1 P1 V C1
= .
C2 P2 V C2

Como V C1 = x, C2 P2 = R e V C2 = R, temos

C1 P1 x
= ∴ C1 P1 = x.
R R
Com isso, a seção do sólido S é a coroa circular entre os cı́rculos de raios x e R, logo, sua área é dada por

SessãoS = πR2 − πx2 = π(R2 − x2 ).

A seção da esfera pelo plano será o cı́rculo de raio CP3 da figura. Como o triângulo OC3 P3 é reto em C3 , temos
2 2 2
OP3 = CP3 + OC3 ,

e, como OC3 = x e OP3 = R,


2
R2 = CP3 + x2 ,
logo
2
CP3 = R2 − x2 .
Com isso, a área da seção da esfera pelo plano será dada por

Sessãoesfera = π(R2 − x2 ) = SessãoS ,

como querı́amos provar.


O raciocı́nio é idêntico para planos acima do vértice e do centro da esfera, bastando alterar a figura.
Para o plano que passa pelo vértice dos cones e pelo centro da esfera, as seções em ambos os sólidos serão cı́rculos de
raio R e, assim, terão mesma área.
(b) Pelo Princı́pio de Cavalieri, a esfera terá o mesmo volume do sólido S. Mas este sólido tem volume dado pelo volume
do cilindro de base de raio R e altura 2R, subtraı́do de dois cones sólidos de base de raio R e altura R. Assim,

π · R2 · R 2πR3 6πR3 − 2πR3


 
4
Volesfera = π · R2 · 2R − 2 = 2πR3 − = = πR3 .
3 3 3 3

Questão 02 [ 2,00 ]

Em um tetraedro ABCD, AB = x, CD = y e as demais arestas medem z. Determine a distância entre as arestas


AB e CD em função de x, y e z.

Solução
Sejam M e N pontos médios de AB e CD, respectivamente.

Os triângulos ACB e ADB são congruentes (LLL) e isósceles, logo, as alturas CM e DM são congruentes. Assim, o triângulo
DM C é isósceles, e, como N é ponto médio de CD, M N será altura de DM C. Da mesma forma, M N é altura do triângulo
isósceles ABN . Com isso, M N é o segmento da perpendicular comum às arestas AB e CD, logo a distância entre estes
segmentos é M N . Vamos então calcular este comprimento.
O triângulo AM D é retângulo em M , logo, denotando h = DM ,
 x 2
z2 = + h2 ,
2
portanto
x2
h2 = z 2 − .
4
Denotando d = M N , como DN M é um triângulo retângulo em N , temos
 y 2
h2 = d 2 + ,
2
logo
 y 2 y2
d 2 = h2 − = h2 − ,
2 4
e então
x2 y2
d2 = z 2 − − .
4 4
Com isso, r
x2 y2
d= z2 − − .
4 4

Questão 03 [ 2,00 ]

BE 1 CF 1
Sobre o lado BC de um quadrado ABCD marcam-se os pontos E e F tais que = e = . Os segmentos
BC 3 BC 4
AF e ED intersectam-se em P . Determine a que fração da área do quadrado ABCD corresponde a área do triângulo
BP E.

Solução
Seja a a medida da aresta do quadrado. Assim,
1 a
BE = BC =
3 3
e
1 a
CF = BC = .
4 4
Como E F̂ P = DÂP , F ÊP = AD̂P (alternos internos nos dois casos) e E P̂ F = DP̂ A (opostos pelo vértice), os triângulos
AP D e F P E são semelhantes. Denotando por h e H as alturas desses triângulos, relativas as lados EF e DA, respectivamente,
temos
h EF
= .
H DA

Como DA = a e
a a 5a
EF = BC − BE − CF = a − − = ,
3 4 12
temos então 5a
h 5
= 12 = ,
H a 12
e, portanto,
5
h= H.
12
Mas h + H = a, logo H = a − h e então
5
h= (a − h),
12
logo
5
h+ h = a,
12
que implica
17 5
h= a
12 12
ou ainda
5
h= a.
17
Assim, a área de BP E é

1 1 a 5 5 2
· BE · h = · ·
S= a= a .
2 2 3 17 102
Como a área de ABCD é a2 , a área do triângulo BP E é 102
5
da área de ABCD.

Questão 04 [ 2,00 ]

Um poliedro convexo com 32 vértices possui apenas faces triangulares. Determine o número de arestas e faces deste
poliedro.

Solução
Vamos denotar por F o número de faces e A o número de arestas. Como este poliedro tem apenas faces triangulares, temos

2A = 3F

(isto é, cada face contabiliza 3 arestas, sendo que, cada aresta é contada duas vezes, uma para cada face em que está contida).
Com isso,
3
A= F.
2
Pelo Teorema de Euler, temos
32 − A + F = 2,

logo
3
32 − F +F =2
2
e, então,
F
− = −30,
2
implicando F = 60.
Com isso, temos
3
A= · 60 = 90.
2

Questão 05 [ 2,00 ]

(a) Usando apenas a identidade fundamental da trigonometria e as fórmulas de arcos duplos prove que: cos(2x) =
2 cos2 (x) − 1, para todo x real.
x
= 13 , calcule cos(x).

(b) Sabendo que cos 2
Solução

(a) Usando a relação fundamental da trigonometria e as fórmulas de arcos duplos temos que:

cos2 (x) + sen2 (x) = 1

e
cos(2x) = cos2 (x) − sen2 (x).

Somando as equações anteriores segue que:


cos(2x) + 1 = 2 cos2 (x).

Logo, cos(2x) = 2 cos2 (x) − 1.

(b) Usando o item (a) temos que:


     2
2x 2x 1 7
cos(x) = cos = 2 cos2 −1=2 =− .
2 2 3 9
MESTRADO PROFISSIONAL EM MATEMÁTICA EM REDE NACIONAL

AVF - MA13 - 2015.1 - Gabarito

Questão 01 [ 2,00 pts ]

Mostre que se uma reta é tangente a um cı́rculo então ela é perpendicular ao raio que liga o centro do cı́rculo ao
ponto de tangência.

Solução
Considere o cı́rculo de centro O e seja T o ponto de tangência da reta r ao cı́rculo. Designemos por P o pé da perpendicular
a reta r baixada por O. Observe que basta mostrar que P = T .

Suponha, por contradição, que P e T são distintos, assim o triângulo OP T é retângulo. Logo OT > OP , pois o maior lado
se opõe ao maior ângulo. Como OT é o raio do cı́rculo, P é um ponto interior do cı́rculo.
′ ′ ′
Seja T ∈ r tal que P T = P T ′ com T ̸= T . Daı́, os triângulos OP T e OP T são congruentes por LAL. Logo OT = OT ′ e

então T pertence ao cı́rculo. Contradição, pois a reta r é tangente ao cı́rculo em T .
Portanto P = T .

Questão 02 [ 2,00 pts ]

Denotando por l1 e l2 as medidas dos lados não paralelos de um trapézio, e por d1 e d2 as distâncias do ponto de
interseção das diagonais a estes lados, respectivamente, mostre que
d1 l2
= .
d2 l1

Solução
Sejam ABCD o trapézio, com lados não paralelos AD e BC medindo l1 e l2 respectivamente, e P o ponto de interseção das
diagonais.
Os triângulos ACD e BCD possuem a mesma base CD e alturas de mesma medida (pois AB e CD são paralelos), logo
possuem a mesma área. Com isso,

Área(AP D) = Área(ACD) − Área(P CD)


= Área(BCD) − Área(P CD) = Área(BP C).

Por outro lado,


l1 · d 1
Área(AP D) =
2
e
l2 · d2
Área(BP C) = ,
2
portanto
l1 · d1 l2 · d2
= .
2 2
Mas isto implica que
l1 · d 1 = l2 · d 2

e, consequentemente,
d1 l2
= .
d2 l1

Questão 03 [ 2,00 pts ]

Em uma folha de papel retangular, de medidas 10cm por 6cm, são feitas as dobras paralelas AB e CD, como na
figura abaixo. Se DE e BF medem ambos 1 cm, determine a medida de AE.

Solução
Seja G tal que EG é o lado da folha retangular que contém o ponto A e denote por G′ o ponto correspondente a G após a
dobra, como na figura abaixo. Seja ainda P a projeção ortogonal de D sobre F G.
Denotando AE = x, teremos então AG = AG′ = 6 − x.

Como P G = DE = BF = 1cm, e como F G = 10cm temos P B = 8cm. E, como DP = EG = 6cm, aplicando o Teorema de
Pitágoras ao triângulo DP B, temos BD = 10cm. Além disso, como BG′ = BG = 9cm, temos então DG′ = 1cm.

Os triângulos DEA e DG′ A serão ambos retângulos, com mesma hipotenusa DA e tais que DE = 1 = DG′ . Estes triângulos
serão, portanto, congruentes. Com isso, x = AE = AG′ = 6 − x, o que implica 2x = 6, logo AE = x = 3cm.

Questão 04 [ 2,00 pts ]

Considere um tetraedro ABCD e sejam M , N , P , Q, R e S os pontos médios das arestas AB, AC, AD, BC, CD e
BD, respectivamente. Prove que o volume do octaedro M N P QRS é metade do volume do tetraedro ABCD.

Solução

O volume do octaedro M N P QRS é obtido retirando-se, do volume do tetraedro ABCD, os volumes dos tetraedros AP M N ,
BM QS, CN RQ e DP RS.

Considere o tetraedro AP M N . Como M , N e P são pontos médios de AB, AC e AD, respectivamente, AP M N é semelhante
1
ao tetraedro ABCD, com razão . Com isso,
2
( )3
V(AP M N ) 1 1
= = .
V(ABCD) 2 8
1
Pelo mesmo raciocı́nio, os tetraedros BM QS, CN RQ e DP RS serão também semelhantes a ABCD com razão , logo
2
V(BM QS) V(CN RQ) V(DP RS) 1
= = = .
V(ABCD) V(ABCD) V(ABCD) 8
Com isso,
1
V(AP M N ) = V(BM QS) = V(CN RQ) = V(DP RS) = V(ABCD),
8
e então

V(M N P QRS) = V(ABCD) − V(AP M N ) − V(BM QS) − V(CN RQ) − V(DP RS)
1
= V(ABCD) − 4 · V(ABCD)
8
1
= · V(ABCD).
2
Questão 05 [ 2,00 pts ]

Determine o cosseno do ângulo entre duas faces de um octaedro regular que possuam uma aresta em comum.

Solução
Sejam A e B dois vértices opostos do octaedro, e M o ponto médio de uma das arestas do octaedro que não tenha A ou B
como extremo, como na figura abaixo.

A medida do ângulo entre as faces será dado por AM̂ B.


Denote por a a medida da aresta do octaedro. Sendo O o centro do octaedro, O será o centro da seção o octaedro destacada
a
na figura, que é um quadrado. Assim, M O = 2
.
O segmento M A será a altura da face que o contém, ou seja, a altura de um triângulo equilátero de lado a. Assim,
√ √
a 3 a 3
MA = 2
. Da mesma forma, M B = 2
.
O triângulo AOM é retângulo em O. Por Pitágoras, temos então
2 2 2
OA + OM = M A ,

que implica
( a )2 ( √ )2
2 a 3
OA + = ,
2 2
e então
2 3a2 a2
OA = − .
4 4
Com isso, √
a 2
OA = .
2
Como OB ≡ OA, temos

AB = OA + OB = 2OA = a 2.
√ √
O triângulo AM B terá então lados de medidas AB = a 2 e AM = BM = a 2 3 . Pela Lei dos Cossenos, temos então
2 2 2
( )
AB = AM + BM − 2 AM BM cos AM̂ B ,

logo
( √ )2 ( a√3 )2 ( a√3 )2 ( √ )2
a 3 ( )
a 2 = + −2 cos AM̂ B ,
2 2 2
e então ( )
3a2 3a2

2a2 = cos AM̂ B .
2 2
Cancelando o a2 e simplificando a igualdade acima, temos
( ) 1
cos AM̂ B = − .
3
MESTRADO PROFISSIONAL EM MATEMÁTICA EM REDE NACIONAL

AVF - MA13 - 2016.1 - Gabarito

Questão 01 [ 2,00 pts ]

Em um triângulo ABC de perı́metro 9, o lado BC mede 3 e a distância entre os pés das bissetrizes interna e externa
partindo de A é igual a 4. Calcule as medidas dos lados AB e AC do triângulo.

Solução

Sejam P e Q os pés das bissetrizes interna e externa, respectivamente, partindo de A. Denotando x = AB, temos AC =
2P − AB − BC = 9 − x − 3 = 6 − x. Pelo Teorema das Bissetrizes Internas,

BP PC BP + P C
= = ,
AB AC AB + AC
logo
BP PC 3
= = .
x 6−x 6
Com isso,
x 6−x
BP = e PC = .
2 2
Como QP = 4, temos
x
QB = QP − BP = 4 −
2
e
6−x
QC = QP + P C = 4 + .
2
Pelo Teorema das Bissetrizes Externas,
QB QC
= ,
AB AC
logo
x 6−x
4− 4+
2 = 2 .
x 6−x
Simplificando,
4 1 4 1
− = + ,
x 2 6−x 2
e então
6 − 2x 1
= .
x(6 − x) 4
Desta forma, temos x2 − 14x + 24 = 0, implicando x = 2 ou x = 12.
Repare, porém, que AB = x = 12 não é possı́vel, pois este valor é maior que o perı́metro do triângulo. Assim, AB = 2, o
que implica AC = 6 − AB = 4.
Esta solução considerou o vértice B entre os pés das bissetrizes relativas a A. Se C estivesse entre os pés das bissetrizes,
terı́amos AB = 4 e AC = 2.
Questão 02 [ 2,00 pts ]

Considere um triângulo ABC retângulo em A. Por A, trace uma reta r perpendicular a BC, que encontra BC em
H. Em r, marque o ponto P , tal que AP = BC, estando A entre P e H. Trace os segmentos P B e P C.
2
BC
(a) Prove que a área do quadrilátero P BAC é .
2
(b) Considere o quadrado ABDE tal que A está entre E e C. Prove que o triângulo BCD é congruente ao triângulo
AP B.
2
AB
(c) Prove que área do triângulo BCD é .
2

Solução

(a) Observe que Área(P BAC) = Área(AP B) + Área(AP C).

Como BH é a altura do triângulo AP B relativa ao lado AP , temos


BH · AP BH · BC
Área(AP B) = =
2 2
e, como CH é a altura do triângulo AP C relativa ao lado AP , temos
CH · AP CH · BC
Área(AP C) = = .
2 2
Assim,
 2
BH · BC CH · BC BC BH + CH BC
Área(P BAC) = Área(AP B) + Área(AP C) = + = = .
2 2 2 2

(b) Temos C ÂH = AB̂C, pois ambos são complementares ao ângulo AĈB, e P ÂE = C ÂH, pois os ângulos são opostos
pelo vértice. Com isso, P ÂE = AB̂C. Assim, P ÂB = C B̂D, pois ambos os ângulos são a soma de AB̂C com um ângulo
reto.
Sabemos também que BC = AP (dado do problema) e BD = AB (pois ABDE é um quadrado), logo, pelo caso LAL,
os triângulos BCD e AP B são congruentes.
(c) Como, pelo item (b), BCD e AP B são congruentes, temos
BH · BC
Área(BCD) = Área(AP B) = ,
2
como calculado no item (a). Por outro lado, uma das relações métricas do triângulo retângulo nos diz que BH · BC =
2
AB , logo
2
AB
Área(BCD) = .
2

Questão 03 [ 2,00 pts ]

OABC é um tetraedro tal que AÔB = AÔC = B ÔC = 90◦ , onde OA = x, OB = y e OC = z. Sabe-se que
√ √ √
AB = 41, AC = 2 13 e BC = 61.

(a) Determine x, y e z.

(b) Determine o volume do tetraedro ABCO.

Solução
(a) Aplicando o Teorema de Pitágoras nos triângulos ABO, ACO e BCO, todos eles retângulos em O, temos:
2
x2 + y 2 = AB = 41. (I)
2
x2 + z 2 = AC = 52. (II)
2
y 2 + z 2 = BC = 41. (III)
Somando membro a membro as três equações anteriores, temos:

2x2 + 2y 2 + 2z 2 = 154

e então
x2 + y 2 + z 2 = 77 (IV)
Das subtrações abaixo, obtemos os valores desejados:

(IV) − (III) ⇒ x2 = 16 ∴ x = 4cm.

(IV) − (II) ⇒ y 2 = 25 ∴ y = 5cm.


(IV) − (I) ⇒ z 2 = 36 ∴ z = 6cm.
(b) Tomando o triângulo ABO como base do tetraedro, OC será sua altura. Daı́, seu volume é:
1 x·y 1 4·5
V = · ·z = · · 6 ∴ V = 20cm3 .
3 2 3 2

Questão 04 [ 2,00 pts ]

Sejam L um cı́rculo de raio 1 de centro O e r uma reta (no mesmo plano que L) exterior a L. Diga como construir,
com régua e compasso, uma reta s tangente ao cı́rculo e que determina, com r, um ângulo de 45◦ .

Solução
A solução abaixo não depende das medidas dadas.
Uma possı́vel construção está representada acima e descrita abaixo:

1. Trace a reta perpendicular a r passando por O e seja P sua interseção com r.

2. Construa o cı́rculo de centro P e raio P O e seja A uma das interseções deste cı́rculo com r. Repare que, por construção,
o triângulo AP O é retângulo em P e isósceles, com OP ≡ AP . Com isso, P ÂO = 45◦ .

3. Trace a reta t determinada por A e O e seja B a interseção de t com o cı́rculo dado, de forma que O esteja entre A e B.

4. Construa o cı́rculo de centro B e raio BA, e seja C seu ponto interseção com r diferente de A. Note que CB ≡ AB,
logo ABC é isósceles, com AĈB = C ÂB = 45◦ . Teremos ainda que AB̂C = 90◦ .

5. A reta s procurada é a determinada por B e C.

Questão 05 [ 2,00 pts ]

Considere três retas r, s e t do espaço tais que qualquer plano seja concorrente a pelo menos uma destas retas.
Considere ainda um poliedro tal que

• todas as suas faces são quadriláteros;

• cada uma de suas arestas é paralela a alguma das retas r, s ou t; e

• se um plano contém uma das faces, nenhum vértice do poliedro pode estar neste plano, além dos vértices da
própria face.

Prove que

(a) todas as faces deste poliedro são paralelogramos, e

(b) este poliedro é um prisma.

Solução
(a) Considere uma das faces do poliedro, a qual chamaremos de ABCD. Sem perda de generalidade, digamos que AB k r.
Não poderemos ter BC k r, pois, neste caso, teremos AB e BC consecutivos e colineares. Digamos então, sem perda de
generalidade, que BC k s.
Observe que já concluı́mos que cada uma das retas r e s é paralela ou está contida no plano da face ABCD. Assim,
pela informação dada sobre as retas, t deve ser concorrente a tal plano, logo não pode ser paralela a este plano nem
pode estar contida nele. Portanto, nenhuma das arestas CD e AD desta face pode ser paralela à reta t.
Não poderemos ter CD k s (senão BC e CD seriam colineares) nem, como já vimos, CD k t. Portanto, CD k r. Da
mesma forma, AD não pode ser paralela a r, logo AD k s.
Até aqui, já vimos que ABCD é um paralelogramo. Mas a mesma argumentação vale para qualquer uma das faces do
poliedro, portanto todas as suas faces são paralelogramos.

(b) Seja ABCD uma das faces. Pelo item (a), podemos supor, sem perda de generalidade, AB e CD paralelas a r e BC e
AD paralelas a s.
Considere uma aresta que tenha A, B, C ou D como vértice, e que não esteja na face ABCD. Chamemos esta aresta
de EA. Como E não pertence ao plano de ABCD, EA não é paralela a r ou s, logo EA k t.
Mas isto vale para todas as arestas que têm um vértice na face ABCD, portanto todas estas arestas são paralelas a t, e
de cada vértice A, B, C e D sai apenas uma aresta que não está na face ABCD.
A face que tem A, B e E como três de seus vértices deverá ser um paralelogramo (item (a)), portando, seu quarto
vértice F é tal que EF k AB e BF k AE.
A face adjacente a ABF E, compartilhando a aresta BF , deverá ser então um paralelogramo BCGF , com F G k BC. A
face adjacente a BCGF , compartilhando a aresta CF , deverá ser então um paralelogramo CDHG, com GH k CD k AB.
Por fim, teremos a face ADHE, com HE k AD k BC.
Repare que EF GH é, então, um paralelogramo, pois EF e GH são paralelos a AB, e F G e EH são paralelos a BC.
Temos ainda que EF GH deverá ser uma face. De fato, se não fosse assim, haveria uma aresta EI, F I, GI ou HI que
não estaria no plano de EF GH, portanto não seria paralela a r e s, e nem poderia ser paralela a t, pois, neste caso,
terı́amos duas arestas colineares.
Assim, temos o prisma ABCDEF GH, tal que ABCD e EF GH são bases paralelas, com AE k BF k CG k DH.
MA13 Geometria I - GABARITO DA AVALIAÇÃO 2 - 2012/2

Questão 1. (pontuação: 2)

No setor AOB de centro O, raio OA = 3 e ângulo AOB = 60o está inscrita uma circunferência como mostra a figura.

a) Calcule o raio dessa circunferência.

b) Calcule a área da região sombreada.

Uma solução

r M
C
r
O D A

a) Seja M o ponto médio do arco AB. O raio OM passa pelo centro C da circunferência inscrita no setor. Seja CD
perpendicular a OA como mostra a figura acima e seja r = CD = DM o raio da circunferência. Como AOM = 30o
então, no triângulo ODC tem-se OC = 2r e, portanto, OM = 3r = 3, ou seja, r = 1 .

b) A área sombreada (S) é igual à área do setor AOM subtraı́da da área do triângulo ODC e da área do setor
DCM do cı́rculo de centro C.
π32 3π
A área do setor AOM é 12 = 4 .
√ OD.OC

3
Como CD = 1 e OC = 2, então OD = 3 e a área do triângulo ODC é 2 = 2 . Por outro lado, o ângulo
o
DCM mede 120 e, portanto, o setor DCM do cı́rculo de centro C tem área igual à terça parte da área do cı́rculo
√ √
π 3π 3 π 5π−6 3
de centro C, ou seja, 3. Assim, a área sombreada é S = 4 − 2 − 3 , ou seja, S = 12 .

Questão 2. (pontuação: 2)

O Teorema das Três Perpendiculares tem o seguinte enunciado:


“A reta r é perpendicular ao plano α no ponto A. A reta s está contida em α e não passa por A. O ponto B da
reta s é tal que AB é perpendicular a s. Então, se P é qualquer ponto de r, P B é perpendicular a s.”

a) Faça uma figura que descreva o enunciado do Teorema.

b) Demonstre o Teorema.
Uma solução
a)

r

P

 A
B

b) Como r é perpendicular a α então r é ortogonal a qualquer reta de α , portanto r é ortogonal a s. Porém,


AB é perpendicular a s. Assim, s é ortogonal a duas retas concorrentes: AB e r. Logo s é perpendicular ao plano
determinado por AB e r, que chamaremos de plano β. Como P e B são pontos de β então s é perpendicular a P B,
como querı́amos demonstrar.

Questão 3. (pontuação: 2)

Em um cubo, ABCD e EF GH são faces opostas e AE, BF , CG e DH são arestas paralelas. Sejam M e N os
pontos médios das arestas BC e DH, respectivamente.

a) Se a aresta do cubo mede 2, calcule a distância entre os pontos M e N .

b) Calcule o cosseno do ângulo entre as retas AB e N M .

Uma solução

H
G

E
N F
O

D
C

 M
A B


a) Observe a figura acima. No triângulo CDN , retângulo em D, CD = 2 e DN = 1. Consequentemente, N C = 5.
2 2 2
Como a aresta BC é perpendicular à face DCGH, o triângulo M CN é retângulo em C. Daı́, M N = N C + M C =

5 + 1 = 6, ou seja M N = 6.
b) Façamos uma translação do segmento M N de forma que o ponto M concida com o ponto B. Nessa translação,
o ponto N coincidirá com o ponto O, centro da face ADHE. O ângulo entre as retas reversas AB e N M é o ângulo
entre as concorrentes AB e OB, ou seja, o ângulo ABO = θ.

No triângulo ABO, temos AB = 2, OB = N M = 6 e AO é a metade da diagonal do quadrado ADHE, ou seja,

AO = 2.
A lei dos cossenos no triângulo ABO fornece:

√ √ √
( 2)2 = 22 + ( 6)2 − 2.2. 6.cos θ

6
Daı́, encontramos cos θ = 3 . Isto também pode ser obtido notando-se que o triângulo ABO é retângulo em A e
usando-se diretamente a definição de cosseno.

Questão 4. (pontuação: 2)

O trapézio ABCD tem bases AB e CD. A altura do trapézio mede 8. As bases medem AB = 10 e CD = 6. As
diagonais AC e BD do trapézio dividiram o trapézio em quatro triângulos. Calcule as áreas dos quatro triângulos
em que o trapézio ficou dividido.

Uma solução

D 6 C
x

P
8

8–x

A B
10

Notação: (XY Z) representa a área do triângulo de vértices X, Y e Z.


Seja P o ponto de interseção das diagonais e seja x a distância de P à base menor do trapézio. Como os triângulos
6 x 6.3 10.5
P AB e P CD são semelhantes temos 10 = 8−x , o que dá x = 3 . Assim, (P CD) = 2 = 9 e (ABC) = 2 = 25.

10.8
(DAP ) = (DAB) − (P AB) = − 25 = 40 − 25 = 15
2

(CP B) = (CAB) − (P AB) = 40 − 25 = 15


As áreas dos quatro triângulos estão na figura abaixo:

15 15

25
Questão 5. (pontuação: 2)

No cubo ABCDA0 B 0 C 0 D0 de aresta a, os pontos M , N , P e Q são médios das arestas A0 B 0 , B 0 C 0 , C 0 D0 e A0 D0 ,


respectivamente. Foram feitas as seções pelos planos AM Q, BN M , CP N e DP Q. Retirando-se os quatro tetraedros
formados, resultou o poliedro P ilustrado na Figura 1. O poliedro P possui duas bases paralelas e faces laterais
triangulares. Ele é um prismatóide.
a) Calcule o volume do poliedro P .

D' P
C'
Q
N
A' M
B'

D
C

A
B
Figura 1

Observe agora a Figura 2; pelo ponto médio X da aresta AA0 foi traçado um plano paralelo à face ABCD que
determinou em P uma seção octogonal. A forma dessa seção equidistante das bases do poliedro P , que é chamada
se seção média, está ilustrada na Figura 3.
No poliedro P , representaremos a área da base ABCD por S, a área da base M N P Q por s, a área da seção média
por Sm e a distância entre as bases por h.

b) Calcule a área da seção média e calcule o volume de P usando a fórmula do volume dos prismatóides: V =
h
6 (S + s + 4Sm ) .

D' P
C'
Q N
A' M
B'

X
D
C

A B X
Figura 2 Figura 3
Uma solução

a
a) Um dos tetraedros retirados é AA´M Q. Sua base é o triângulo retângulo A´M Q de catetos A´M = A´Q = 2 e
3
altura AA´= a . O volume desse tetraedro é 13 . (a/2)(a/2)
2 .a = a
24 . Como quatro desses tetraedros foram retirados, o
3 a3 3 a3 5a3
volume do poliedro P é V = a − 4. 24 = a − 6 = 6 .

b) A seção média é obtida de um quadrado XY ZW , congruente com ABCD retirando-se quatro triângulos
retângulos isósceles congruentes. O plano da seção média corta a aresta BB´do cubo em Y e corta as arestas M A e
M B do poliedro P em E e F , respectivamente.

X E a/2 F Y

AB a
Temos EF = 2 = 2 (já que a reta XY une pontos médios de lados do triângulo AM B) e, consequentemente,
a2
XE = F Y = a4 . Assim, cada um dos pequenos triângulos retângulos tem área 12 . a4 . a4 = 32 e a área da seção média é
2 2
2
Sm = a − 4. a32 = 7a8 .
Aplicando a fórmula do volume do prismatóide temos:

a 2 a2 7a2 a a2 7a2 a 10a2 5a3


V =
(a + + 4. ) = (a2 + + )= . =
6 2 8 6 2 2 6 2 6
o que coincide com o resultado do item a).
GABARITO MA13 Geometria I - Avaliação 3 - 2012/2

A área de um triângulo ABC será denotada por (ABC).

Questão 1. (pontuação: 2)

A figura abaixo mostra as semirretas perpendiculares r e s, três circunferências pequenas cada uma com raio igual
a 1 e uma circunferência grande de centro O. Uma das circunferências pequenas é tangente a r e a s, cada uma das
outras duas é tangente a ela e a uma das semirretas, e a circunferência grande é tangente às semirretas e a duas das
circunferências pequenas.

Calcule o raio da circunferência grande.

Uma solução:
Na figura a seguir, A é o centro da circunferência pequena que tangencia r e a circunferência grande, OT é
perpendicular a r e a reta BC passa por A e é paralela a r.

s R

C B
A
r T
Seja R o raio da circunferência grande. No triângulo retângulo ABO temos OA = R+1, OB = R−1 e AB = R−3.
O teorema de Pitágoras conduz à equação R2 − 10R + 9 = 0 cujas raı́zes são 1 e 9.
Devido às caracterı́sticas do problema, a menor raiz é o raio da circunferência pequena tangente às duas semirretas
e a maior raiz é o raio da circunferência grande.
O raio da circunferência grande é igual a 9.

Questão 2. (pontuação: 2)

No triângulo ABC a bissetriz do ângulo BAC encontra o lado BC em D.


DB AB
a) Prove que DC = AC (teorema da bissetriz interna).

b) Use o teorema acima e a figura abaixo para calcular a tangente de 15o .

15o x
15o

Uma solução:
a) Se dois triângulos têm mesma altura, então a razão entre suas áreas é igual à razão entre suas bases. Assim,

(ADB) DB
=
(ADC) DC

E d
d
B D C
Sejam DE e DF perpendiculares a AB e AC como na figura anterior. Como todo ponto da bissetriz de um ângulo
equidista dos lados desse ângulo, então DE = DF = d. Assim,

(ADB) (1/2).AB.d AB DB AB
= = , portanto = , c.q.d.
(ADC) (1/2).AC.d AC DC AC

b) Como o triângulo ABC da figura é retângulo em B e tem ângulo B ÂC = 30o e hipotenusa AC = 2, então

BC = 1 e AB = 3.

1–x
2
D
15o x
15o
A 3 B


DB AB x 3
O teorema da bissetriz interna aplicado a esse triângulo fornece: DC = AC , ou seja, 1−x = 2 .
Aplicando propriedades das proporções podemos escrever:

x 1−x x + (1 − x) 1
√ = = √ = √ .
3 2 3+2 2+ 3
Porém, observando o triângulo ABD, vemos que √x3 é a tangente do ângulo de 15o . Assim,

1 √
tan15o = √ = 2 − 3.
2+ 3

Questão 3. (pontuação: 3)

O losango ABCD tem lado 3 e ângulo  = 60o . Os pontos M , N , P e Q pertencem aos lados AB, BC, CD e DA,
respectivamente e são tais que AM = BN = CP = DQ = 1 .

a) Justifique, de forma breve, porque o quadrilátero M N P Q é um paralelogramo.

b) Calcule a área do quadrilátero M N P Q.

c) Calcule a distância entre os pontos M e P .


Uma solução:

D
Q
P
A C
M
N
B

a) Os triângulos AM Q e CP N são congruentes (caso LAL). Daı́, M Q = N P . Os triângulos BN M e DQP são con-
gruentes (caso LAL); daı́, M N = QP . Assim, o quadrilátero M N P Q possui dois pares de lados opostos congruentes.
Logo, é um paralelogramo.
√ √
3 3
b) (AM Q) = 21 .AM.AQ.sen 60o = 12 .1.2. 2 = 2 .
Como os ângulos de 60o e 120o possuem mesmo seno, concluı́mos que os triângulos AM Q, BN M , CP N e DQP

3
possuem todos a mesma área, igual a 2 . √ √
3 9 3
A área do losango é igual a AB.AD.sen 60o = 3.3. 2 = 2 .
A área do paralelogramo é √ √ √
9 3 3 5 3
S= −4 = .
2 2 2

c) Seja R o ponto médio de P D. Como AM é paralelo a DR e ambos têm comprimento 1 então AM RD é um


paralelogramo e M R = AD = 3. Além disso, RP = 1 e M R̂P = 120o .

D
R
P
A C
M

No triângulo M RP a lei dos cossenos fornece:

1 √
M P 2 = M R2 + RP 2 − 2.M R.RP.cos 120o = 32 + 12 − 2.3.1.(− ) = 13 =⇒ M P = 13.
2
Questão 4. (pontuação: 1)

O icosaedro regular é o poliedro formado por 20 faces triangulares equiláteras. Determine quantas diagonais do
icosaedro não passam pelo seu centro.

Uma solução:

O icosaedro possui 20 faces triangulares. Como cada aresta é lado de exatamente duas faces, o número de arestas
20.3
do icosaedro é A = 2 = 30.

O número de vértices pode ser calculado pela relação de Euler V − A + F = 2, de onde V = 12.
Cada segmento que une dois vértices do icosaedro ou é aresta ou é diagonal. Assim, denotando por CV2 o número
de escolhas de subconjuntos com dois elementos do conjunto de todos os vértices do icosaedro, o número de diagonais
do icosaedro é

12!
D = CV2 − A = C12
2
−A= − 30 = 66 − 30 = 36
10!2!

O icosaedro possui 6 pares de vértices diametralmente opostos e cada diagonal que une dois vértices diametralmente
opostos passa pelo centro do icosaedro. Essas são as únicas diagonais que passam pelo centro. Então, o número de
diagonais que não passam pelo centro é 36 − 6 = 30.

Questão 5. (pontuação: 2)

Considere o paralelepı́pedo retângulo de bases ABCD e EF GH e com arestas laterais AE, BF , CG e DH. As
medidas são AB = 6, AD = AE = 4 e M é o ponto médio da aresta EF . São feitas as seções pelos planos M HA e
M BG. Retirando-se os tetraedros EM HA e F M BG resulta o poliedro P.

a) Faça um desenho do poliedro P e calcule seu volume.

b) Determine o cosseno do ângulo entre as retas AH e M G.


Uma solução:

H
G
E M
F
D
C
A
B

O desenho de P está acima.


Se pensarmos o tetraedro EM HA com base EM H e altura EA, podemos calcular seu volume do seguinte modo:

1 EM.EH 1 3.4
v=. .EA = . .4 = 8
3 2 3 2
O tetraedro F M BG tem também volume v = 8 porque é congruente com EM HA.
O volume de P é o volume do paralelepı́pedo subtraı́do dos volumes dos tetraedros, ou seja,

V = 6.4.4 − 2.8 = 96 − 16 = 80

b) O ângulo entre as retas AH e M G é o ângulo entre BG e M G, ou seja, o ângulo θ = B ĜM . Como F M = 3 e



F B = F G = 4 temos, pelo teorema de Pitágoras, BG = 4 2 e M B = M G = 5.

H
G
5
M θ
E
F 22

5 N
D
C
A
B

O triângulo M BG é isósceles. Então, assinalando o ponto N médio do lado BG, temos que M N é perpendicular
a BG. Assim, no triângulo M N G,

NG 2 2
cos θ = = .
MG 5
GABARITO MA13 - Avaliação Final 1o semestre - 2013

Questão 1. (pontuação: 2)

No triângulo isósceles ABC tem-se AB = AC. Os pontos M , N e P dos lados AB, BC e CA são tais que P M = P N .
Sendo P M̂ A = α, N P̂ C = β e M N̂ B = θ mostre que

α+β
θ=
2

Uma solução:

Sejam AB̂C = AĈB = x e P M̂ N = P N̂ M = y.


O ângulo AM̂ N é externo do triângulo M BN . Logo, α + y = x + θ.
O ângulo B N̂ P é externo do triângulo P N C. Logo, β + x = y + θ.
α+β
Somando, temos α + β = 2θ, ou seja, θ = 2 , cqd.

Questão 2. (pontuação: 2)

Considere o triângulo ABC, retângulo em A, sendo BC = a e AC = b. Seja K1 a circunferência de centro C que


passa por A. A circunferência K2 tem centro P sobre o lado BC, é tangente externamente à K1 e é tangente ao lado
AB.

a) (1,0) Descreva como se pode determinar com régua e compasso o ponto P .

b) (1,0) Determine o raio da circunferência K2 em função de a e b.

Obs.: os itens acima podem ser resolvidos de maneira independente.


Uma solução:
a)

Seja D o ponto onde K1 corta BC. A perpendicular a BC por D é tangente a K1 e corta AB em E. A bissetriz
do ângulo DÊB corta BC em P . De fato, pela construção acima, P é equidistante das retas ED e EB. Logo, a
circunferência de centro P que passa por D é tangente a K1 e ao lado AB.

b)

Tracemos P G, perpendicular a AB e P F perpendicular a AC como na figura acima. Sejam P D = P G = x. Da


semelhança dos triângulos CF P e CAB temos:

b−x b
=
b+x a
Assim,

b(a − b)
x=
a+b
.
Questão 3. (pontuação: 2)

A figura a seguir mostra duas semicircunferências com mesmo centro O e com raios OD = r e OA = 2r. Na semi-
circunferência maior foi assinalado um ponto B e ângulo AÔB mede α radianos. O raio OB cortou a circunferência
menor em C e a região R é a que está sombreada (delimitada pelo arco AB, segmento BC, arco CD e segmento
DA) na figura.

a) (1,0) Calcule o perı́metro de R em função de r e α.

b) (1,0) Calcule a área de R em função de r e α.

Uma solução:
a) O comprimento de um arco de circunferência é igual a medida do ângulo central em radianos multiplicada pelo
raio. Assim o perı́metro de R é:

P = 3r + α.2r + r + (π − α)r = (α + π + 4)r

αr 2
b) A área de um setor de ângulo central α em radianos em um cı́rculo de raio r é 2 . Assim a área da região R é:

α(2r)2 (π − α)r2 (3α + π)r2


A= + =
2 2 2

Questão 4. (pontuação: 2)

A aresta da base de uma pirâmide reta de base quadrada mede 2 unidades e a esfera inscrita nessa pirâmide tem
raio r (0 < r < 1).
E

D
C
A
B

a) (1,0) Calcule o volume da pirâmide em função de r.

b) (1,0) Se, para cada valor de r (0 < r < 1 ), o volume da pirâmide é V (r), faça um esboço do gráfico dessa
função.

Uma solução:

a)

Seja O o centro da base ABCD da pirâmide de vértice E como mostra a figura ao lado. Seja M o ponto médio
da aresta BC. Seja K o ponto sobre a altura OE o centro da esfera inscrita na pirâmide. Assim, traçando KT
perpendicular à face EBC temos KO = KT = r. Seja h = OE a altura da pirâmide. Da semelhança dos triângulos
ET K e EOM temos

KT KE
=
OM EM
ou seja,

r h−r
=√
1 h2 + 1
2r
Dessa relação determinamos a altura da pirâmide h = 1−r 2 .
O volume da pirâmide é
1 2 2r 8r
V = .2 . =
3 1 − r2 3(1 − r2 )
.
b) A função que associa r e V é crescente. Quando r se aproxima de 0, temos que V se aproxima de 0. Porém,
quando r se aproxima 1 temos que V tende a +∞. Logo, o gráfico de V (r) tem o seguinte aspecto:

Questão 5. (pontuação: 2)

Um copo de plástico rı́gido e espessura muito fina tem a forma de um tronco e cone com 8 cm de diâmetro na
boca, 6 cm de diâmetro no fundo e 12 cm de altura.

a) (1,0) Determine um valor aproximado para o volume do copo (ou seja, o número inteiro de cm3 que melhor
aproxima o volume).

b) (1,0) Determine um valor aproximado para a área externa total do copo (ou seja, o número inteiro de cm2 que
melhor aproxima a área externa).

Uma solução:
a) Os raios das bases são 4 cm, e 3 cm e a fórmula do volume do tronco de cone fornece o resultado:

πh π.12
V = .(R2 + r2 + Rr) = (16 + 9 + 12) ≈ 3, 14.4.37 ≈ 465 cm3
3 3

b) A geratriz do tronco de cone é igual a 122 + 1 que é aproximadamente igual a 12. Nesse copo, a altura é quase
igual à geratriz.
A área do copo é a soma da área lateral com a área da base, isto é:

A = π(R + r)g + πr2 ≈ 3, 14.[(4 + 3).12 + 32 ] ≈ 292 cm2

Anda mungkin juga menyukai